Leadership and Management Questions

Réussis tes devoirs et examens dès maintenant avec Quizwiz!

Verbal communication includes texting and email. True or False

True

Nurses should know ___. a. the source of healthcare policies b. the content of healthcare policies c. the consequences of healthcare policies d. the role of the professional in providing feedback to policy makers when policies are no longer effective e. All of the above are true.

e. All of the above is true.

At the closure of negotiations the following should have occurred ___. a. new positions should be clarified and well-defined b. trade-offs noted c. bargaining trades clearly articulated d. both sides dissatisfied with the results e. a, b, and c f. All of the above are true.

e. a, b, and c

Healthcare reform involves ___. a. move from a volume service framework to a value-driven model b. a change to user-driven healthcare service c. increasing the use of digital technology d. decreasing the number of people on Medicare e. a, b, and c f. All of the above are true.

e. a, b, and c

Insured individuals include ____. a. those with employer-sponsored health plans b. older citizens c. military d. illegal aliens e. a, b, and c

e. a, b, and c

Maintaining one's competence as a professional nurse includes ___. a. role b. licensure c. technology d. completing a graduate degree e. a, b, and c f. All of the above is true.

e. a, b, and c

Material supply levels are impacted by ___. a. types of services provided b. available staff c. access to supplies d. cost of supplies e. a, c, and d f. All of the above

e. a, c, and d

The following is true about teams ___. a. Teams can't be constructed simply because people want to meet or work together. b. Teams are purposeful. c. Accountable teams are relational bodies. d. The work of teams is the sum total of its members contributions. e. a, b, and c f. All of the above are true.

f. All of the above are true.

<p>Organizations providing healthcare services are required to have policies specific to ___.</p> <p>a. provider roles</p> <p>b. patient care procedures</p> <p>c. safety practices</p> <p>d. equipment management</p> <p>e. staffing and scheduling</p> <p>f. All of the above.</p>

f. All of the above.

The delegation can be improper if any of the five rights are violated. These include ___. a. right task b. right patient c. right circumstance d. right directions e. right supervision f. All of these are rights.

f. All of these are rights.

Sullivan and Decker (1997) define change as: A. Planned and purposeful B. Voluntary and carries intrinsic or extrinsic rewards C. Making something different from what it is D. voluntary and for one's own reasons

CorrectC. Making something different from what it is

The provision of guidance, direction, evaluation, and follow-up by a licensed nurse for tasks provided by an CNA (certified nurses aide) is: A. Delegation B. Accountability C. Supervision D. Authority

CorrectC. Supervision

The study of healthcare policies includes ___. a. national or state legislation initiatives and processes b. organizational policies necessary for order and progress c. external agencies d. All of the above are true.

d. All of the above are true.

Maintaining high quality-of-work-life environments requires the commitment of: A. Maintenance, housekeeping, and other service departments B. Leaders and employees of the organization C. Human and financial resource departments D. Owners and stockholders of a company

CorrectB. Leaders and employees of the organization

State the 5 steps of decision making in order

1.Identify the need for a decision 2.Determine the goal or outcome 3.Identify alternatives or actions, and list the benefits and consequences of each 4.Choose an alternative 5.Evaluate the alternative chosen

3. Basic critical thinking

A nurse has been working on a surgical unit for 3 weeks. A patient requires a Foley catheter to be inserted, so the nurse reads the procedural manual for the institution to review how to insert it. The level of critical thinking that the nurse is using is: 1. Commitment 2. Scientific method 3. Basic critical thinking 4. Complex critical thinking

A coach is ___. a. a wise and trusted advisor who guides others on a particular journey. b. one who assists others to develop viable solutions, prioritize them, and then act on them. c. a unit manager or leader. d. None of the above.

b. one who assists others to develop viable solutions, prioritize them, and then act on them.

You are very excited about your new job but you have been late for work several times because of the traffic. At this time you believe the nurse manager will implement a disciplinary action. Which action will the nurse manager most likely take first? Answers: A. Termination B. Written warning C. Suspension D. Verbal warning

D. Verbal warning

1. Smoking is prohibited around oxygen. 3. Do not use electrical equipment around oxygen. 4. Special precautions may be required when traveling with oxygen.

To ensure the safe use of oxygen in the home by a patient, which of the following teaching points does the nurse include? (Select all that apply.) 1. Smoking is prohibited around oxygen. 2. Demonstrate how to adjust the oxygen flow rate based on patient symptoms. 3. Do not use electrical equipment around oxygen. 4. Special precautions may be required when traveling with oxygen.

4. Anticipated when to make choices without others' assistance.

Which of the following is unique to the commitment level of critical thinking? 1. Weighs benefits and risks when making a decision. 2. Analyzes and examine choices more independently. 3. Concrete thinking. 4. Anticipated when to make choices without others' assistance.

The nurse preceptor asked a new graduate nurse to explain the primary difference between the nursing process, and the problem-solving and decision-making processes? The new graduate would be correct in stating that: A. The nursing process applies to patient situations or problems B. The nursing process works only for nurses C. The problem-solving and decision-making processes require group decision making D. The problem-solving and decision-making processes apply to all other problems except patient-centered ones

Correct A. The nursing process applies to patient situations or problems

Practice breakdown is ____. a. disruption or absence of any of the aspects of good practice b. brief excursions from an established boundary for a therapeutic purpose c. a process that obscures the ethical dimensions of a decision d. when a person knows what is right and doesn't want to do it

a. disruption or absence of any of the aspects of good practice

State the three common types of manager leadership styles. DO IN ALPHABETICAL ORDER

Authoritarian, Democratic, Laissez-faire

1. Moving from an acute illness to a health promotion, illness prevention model

Health care reform will bring changes in the emphasis of care. Which of following models is expecting from health care reform? 1. Moving from an acute illness to a health promotion, illness prevention model 2. Moving from illness prevention to a health promotion model 3. Moving from an acute illness to a disease management model 4, Moving from a chronic care to an illness prevention model

3. Ask the family to stay with the patient if possible. 4. Inform the family of the risks associated with side rail use. 6. Discuss alternatives with the family that are appropriate for this patient.

The family of a patient who is confused and ambulatory insists that all four side rails be up when the patient is alone. What is the best action to take in this situation? (Select all that apply.) 1. Contact the nursing supervisor. 2. Restrict the family's visiting privileges. 3. Ask the family to stay with the patient if possible. 4. Inform the family of the risks associated with side rail use. 5. Thank the family for being conscientious and put the four rails up. 6. Discuss alternatives with the family that are appropriate for this patient.

During a hiring interview, which response by a nursing applicant should indicate that the applicant operates from an ethical egoism framework? A. "I would want to be treated in a caring manner if I were mentally ill." B. "This job will pay the bills, and the workload is light enough for me." C. "I will be happy caring for the mentally ill. Working in Med/Surg kills my back." D. "It is my duty in life to be a psychiatric nurse. It is the right thing to do."

ANS: B The applicant's comment reflects an ethical egoism framework. This framework promotes the idea that decisions are made based on what is good for the individual and may not take the needs of others into account.

Northern European Americans value punctuality, hard work, and the acquisition of material possessions and status. A nurse should recognize that these values may contribute to which form of psychopathology? A. Dissociative disorders B. Alzheimer's dementia C. Stress-related disorders D. Schizophrenia-spectrum disorders

ANS: C The nurse should correlate Northern European American values, such as punctuality, hard work, and acquisition of material possessions, with stress-related disorders. Psychopathology may occur when individuals fail to meet the expectations of the culture.

When asked about teleology, a nurse educator may provide an example of teleology such as: A. "The achievement of a good outcome justifies using less than desirable means to attain it." B. "Do unto others as you would have them do unto you." C. "Murder is always wrong." D. "A person's character must be developed so that he will be predisposed to behave virtuously."

CorrectA. "The achievement of a good outcome justifies using less than desirable means to attain it."

A new staff nurse is hired as a full-time employee who works 40 hours a week. The nurse would be referred to as which of the following? A. 1.0 FTE B. 0.5 FTE C. 0.9 FTE D. 0.8 FTE

CorrectA. 1.0 FTE

During a class on cultural beliefs, the instructor would most likely explain that these beliefs are: A. Hereditary B. Reality C. Conscious and unconscious D. Genetic

CorrectC. Conscious and unconscious

The nurse manager is trying to plan the shifts in the most effective manner. The manager knows that one characteristic of effective shift planning includes which of the following? A. Nobody died B. Everyone showed up for work C. Evaluation of optimal and reasonable outcomes D. Getting the job done in the least amount of time

CorrectC. Evaluation of optimal and reasonable outcomes

2. Since your husband has died, what have you been doing in the evening when you feel lonely?

During the assessment interview of an older woman experiencing a developmental crisis, the nurse asks which of the following questions? 1. How is this flood affecting your life? 2. Since your husband has died, what have you been doing in the evening when you feel lonely? 3. How is having diabetes affecting your life? 4. I know this must be hard for you. Let me tell you what might help.

2. Pulling the curtain to provide privacy 3. Offering to discuss information about her condition 5. Sitting quietly by her bed and hold her hand

Your patient has just been told that she has cancer, and she is crying. Which actions facilitate therapeutic communication? (Select all that apply.) 1. Turning on the television to her favorite show 2. Pulling the curtain to provide privacy 3. Offering to discuss information about her condition 4. Asking her why she is crying 5. Sitting quietly by her bed and hold her hand

Challenges with technology include ____. a. computer and technology competency b. identification of the boundaries between employer and personal electronic products c. use of social media d. All of the above are true.

d. All of the above are true.

Supervision ___. a. is the provision of guidance or direction, evaluation, and follow-up by the delegator for accomplishment of a task delegated to another. b. is to watch over a particular activity or task being carried out by people and ensure that it is carried out correctly. c. is the transfer of responsibility for the performance of a selected nursing activity or task from a licensed nurse authorized to perform the activity or task to someone who does not have the authority. d. a and b

d. a and b

1. "Are you thinking of suicide?"

A 34-year old man who is anxious, tearful, and tired from caring for his three young children tells you that he feels depressed and doesn't see how he can go on much longer. Your best response would be which of the following? 1. "Are you thinking of suicide?" 2. "You've been doing a good job raising your children. You can do it!" 3. "Is there someone who can help you?" 4. "You have so much to live for."

1. A safe environment promotes patient activity.

A 62-year old woman is being discharged home with her husband after surgery for a hip fracture from a fall at home. When providing discharge teaching about home safety to his patient and her husband, the nurse knows that: 1. A safe environment promotes patient activity. 2. Assessment focuses on environmental factors only. 3. Teaching home safety is difficult to do in the hospital setting. 4. Most accidents in the older adult are caused by lifestyle factors.

4. Clear the area around the child to protect child from injury.

A child in the hospital starts to have a grand mal seizure while playing in the playroom. What is your most important nursing intervention in this situation? 1. Begin cardiopulmonary respiration. 2. Restrain the child to prevent injury. 3. Place a tongue blade over the tongue to prevent aspiration. 4. Clear the area around the child to protect child from injury.

2. "Tell me what bothers you the most about this experience."

A child who has been in a house fire comes to the emergency department with her parents. The child and parents are upset and tearful. During the nurse's first assessment for stress the nurse says: 1. "Tell me who I can call to help you." 2. "Tell me what bothers you the most about this experience." 3. "I'll contact someone who can help get you temporary housing." 4. "I'll sit with you until other family members can come help you get settled."

4. Automobile accidents, suicide, and substance abuse

A couple is with their adolescent daughter for a school physical and state they are worried about all the safety risks affecting this age. What is the greatest risk for injury for an adolescent? 1. Home accidents 2. Physiological changes of aging 3. Poisoning and child abduction 4. Automobile accidents, suicide, and substance abuse

3. An adventitious crisis

A grandfather living in Japan worries about his two young grandsons who disappeared after a tsunami. This is an example of: 1. A situational crisis 2. A maturational crisis 3. An adventitious crisis 4. A developmental crisis

1. Cognitive

A nurse is teaching a 27-year old gentleman how to adjust his insulin dosages based on his blood sugar results. What type of learning is this? 1. Cognitive 2. Affective 3. Adaptation 4. Psychomotor

3. Call the poison control center.

A parent calls the pediatrician's office frantic about the bottle of cleaner that her 2-year old son drank. Which of the following is the most important instruction the nurse gives to the parent? 1. Give the child milk. 2. Give the child syrup of ipecac. 3. Call the poison control center. 4. Take the child to the emergency department.

4. Implementation

A patient in the emergency department has developed wheezing and shortness of breath. The nurse gives the ordered medicated nebulizer treatment now and in 4 hours. Which standard of practice performed? 1. Planning 2. Evaluation 3. Assessment 4. Implementation

3. Provide only the information that the patient needs to go home

A patient newly diagnosed with cervical cancer is going home. The patient is avoiding discussion of her illness and postoperative orders. What is the nurse's best plan in teaching this patient? 1. Teach the patient's spouse 2. Focus on knowledge the patient will need in a few weeks 3. Provide only the information that the patient needs to go home 4. Convince the patient that learning about her health is necessary

4. With silence.

A patient newly diagnosed with type 2 diabetes says, "My blood sugar was just a little high. I don't have diabetes." The nurse responds: 1. "Let's talk about something cheerful." 2. "Do other members of your family have diabetes?" 3. "I can tell that you feel stressed to learn that you have diabetes." 4. With silence.

3. Obtains an interpreter to facilitate communication of medication information

A patient with limited English proficiency is going to be discharged on new medication. How does the nurse complete the discharge teaching? 1. Uses a dictionary to give directions for medication administration 2. Explains the directions to the patient's 14-year old daughter 3. Obtains an interpreter to facilitate communication of medication information 4. Uses a picture board and visual aids to communicate medication administration information

4. Nurses frequently experience stress with the rapid changes in health care technology and organizational restructuring.

A staff nurse is talking with the nursing supervisor about the stress that she feels on the job. The supervising nurse recognizes that: 1. Nurses who feel stress usually pass the stress along to their patients. 2. A nurse who feels stress is ineffective as a nurse and should not be working. 3. Nurses who talk about feeling stress are unprofessional and should calm down. 4. Nurses frequently experience stress with the rapid changes in health care technology and organizational restructuring.

The nurse manager has implemented the process of progressive discipline with a nurse who has violated hospital policy. The nurse manager recognizes that two criteria for corrective actions to be effective are that the actions are: Answers: A. Consistent and impartial B. Pejorative and sound C. Assertive and pertinent D. Measurable and administered by HR

A. Consistent and impartial

A community health nurse is planning a health fair at a local shopping mall. Which middle-class socioeconomic cultural group should the nurse anticipate would most value preventive medicine and primary health care? A. Northern European Americans B. Native Americans C. Latino Americans D. African Americans

ANS: A The community health nurse should anticipate that Northern European Americans, especially those who achieve middle-class socioeconomic status, place the most value on preventative medicine and primary health care. This value is most likely related to this group's educational level and financial capability. Many members of the Native American, Latino American, and African American subgroups value folk medicine practices.

A Latin American man refuses to acknowledge responsibility for hitting his wife, stating instead, "It's the man's job to keep his wife in line." Which cultural belief should a nurse associate with this client's behavior? A. Families are male dominated with clear male-female role distinctions. B. Religious tenets support the use of violence in a marital context. C. The nuclear family is female dominated and the mother possesses ultimate authority. D. Marriage dynamics are controlled by dominant females in the family.

ANS: A The nurse should associate the cultural belief that families are male dominated with clear male-female role distinctions with the client's abusive behavior. The father in the Latin American family usually possesses the ultimate authority.

Which cultural considerations should a nurse identify with Western European Americans? A. They are present-time oriented and perceive the future as God's will. B. They value youth, and older adults are commonly placed in nursing homes. C. They are at high risk for alcoholism due to a genetic predisposition. D. They are future oriented and practice preventive health care.

ANS: A The nurse should identify that most Western European Americans are present oriented and perceive the future as God's will. Older adults are held in positions of respect and are often cared for in the home instead of nursing homes.

During the first interview with a man from Syria who has just lost his son in a car accident, in sympathy for the man's loss, the female nurse reaches out and hugs him. Which is an accurate evaluation of the nurse's action? A. The nurse's action should be evaluated as unacceptable due to breech of cultural norms. B. The nurse's action should be evaluated as empathetic; encouraging expressions of feelings. C. The nurse's action should be evaluated as the technique of offering self. D. The nurse's action should be evaluated as inappropriate due to poor timing.

ANS: A The nurse's action should be evaluated as unacceptable due to breech of cultural norms. During communication, Arab Americans stand close together, maintain steady eye contact, and may touch the other's hand or shoulder but only between members of the same sex.

A female nurse is caring for an Arab American male client. When planning effective care for this client, the nurse should be aware of which of the following cultural considerations? (Select all that apply.) A. Limited touch is acceptable only between members of the same sex. B. Conversing individuals of this culture stand far apart and do not make eye contact. C. Devout Muslim men may not shake hands with women. D. The man is the head of the household and women take on a subordinate role. E. Men of this culture are responsible for the education of their children.

ANS: A, C, D When planning effective care for this client, the nurse should be aware that limited touch within this culture is acceptable only between members of the same sex, that devout Muslim men may not shake hands with women, and that women are subordinate to the man, who is the head of household. Conversing individuals of this culture stand close together and maintain eye contact. Arab American women are responsible for the education of the children.

The experience of being physically restrained can be traumatic. Which nursing intervention would best help the client deal with this experience? A. Administering a tranquilizing medication before applying the restraints B. Talking to the client at brief but regular intervals while the client is restrained C. Decreasing stimuli by leaving the client alone most of the time D. Checking on the client infrequently, in order to meet documentation requirements

ANS: B Restraints are never to be used as punishment or for the convenience of the staff. Connecting with the client by maintaining communication during the period of restraint will help the client recognize this intervention as a therapeutic treatment versus a punishment.

Group therapy is strongly encouraged, but not mandatory, on an inpatient psychiatric unit. The unit manager's policy is that clients can make a choice about whether or not to attend group therapy. Which ethical principle does the unit manager's policy preserve? A. Justice B. Autonomy C. Veracity D. Beneficence

ANS: B The unit manager's policy regarding voluntary client participation in group therapy preserves the ethical principle of autonomy. The principle of autonomy presumes that individuals are capable of making independent decisions for themselves and that health-care workers must respect these decisions.

To effectively care for Asian American clients, a nurse should be aware of which cultural norm? A. Obesity and alcoholism are common problems. B. Older people maintain positions of authority within the culture. C. "Tai" and "chi" are the fundamental concepts of Asian health practices. D. Asian Americans are likely to seek psychiatric help.

ANS: B To effectively care for clients of the Asian American culture, the nurse should be aware that older people in this culture maintain positions of authority. Obesity and alcoholism are low among Asian Americans. The balance of "yin" and "yang," not "tai" and "chi," is the fundamental concept of Asian health practices. In the Asian culture, psychiatric illness is often believed to be out-of-control behavior and would be considered shameful to individuals and families.

When interviewing a client of a different culture, which of the following questions should a nurse consider asking? (Select all that apply.) A. Would using perfume products be acceptable? B. Who may be expected to be present during the client interview? C. Should communication patterns be modified to accommodate this client? D. How much eye contact should be made with the client? E. Would hand shaking be acceptable?

ANS: B, C, D, E When interviewing a client from a different culture, the nurse should consider who might be with the client during the interview, modifications of communication patterns, amount of eye contact, and hand-shaking acceptability. Given that cultural influences affect human behavior, its interpretation, and another person's response, it is important for nurses to understand the effects of these cultural influences to work effectively with diverse populations.

A nurse is preparing to establish a therapeutic relationship with a grieving family from China. Which nursing intervention would be considered most appropriate? A. Touch each member lightly as this enhances the communication process. B. Direct questions to the young males of the family as they maintain positions of authority. C. Avoid direct eye contact as it implies rudeness. D. Remain objective and empathetic as Asians express feelings freely.

ANS: C In the Asian culture, eye contact is often avoided as it connotes rudeness and lack of respect.

There is one bed available on an inpatient psychiatric unit. For which client should a nurse advocate emergency commitment? A. An individual who is persistently mentally ill and evicted from an apartment B. An individual treated in the emergency department (ED) for generalized anxiety disorder C. An individual who is delusional and has a plan to kill his wife D. An individual who rates mood 4/10 and is participating in a no-harm safety plan

ANS: C The criteria for involuntary emergency commitment include danger to self and/or others. Of the four clients considered, the client who is delusional and has a plan to kill his wife meets this criterion as a danger to others.

A brother calls to speak to his sister who has been admitted to the psychiatric unit. The nurse connects him to the community phone and the sister is summoned. Later the nurse realizes that the brother was not on the client's approved call list. What law has the nurse broken? A. The National Alliance for the Mentally Ill Act B. The Tarasoff Ruling C. The Health Insurance Portability and Accountability Act D. The Good Samaritan Law

ANS: C The nurse has violated the Health Insurance Portability and Accountability Act (HIPAA) by revealing that the client had been admitted to the psychiatric unit. The nurse should not have provided any information without proper consent from the client.

A nursing instructor is presenting content on the provisions of the nurse practice act as it relates to their state. Which student statement indicates a need for further instruction? A. "The nurse practice act provides a list of definitions of important terms including the definition of nursing." B. "The nurse practice act lists education requirements for licensure and reciprocity." C. "The nurse practice act contains detailed statements that describe the scope of practice for registered nurses (RNs)." D. "The nurse practice act lists the general authority and powers of the state board of nursing."

ANS: C The nurse practice act contains broad, not detailed, statements that describe the scope of practice for various levels of nursing (APN, RN, LPN), not just for the RN. This student statement indicates a need for further instruction.

Which situation reflects the ethical principle of veracity? A. A nurse provides a client with outpatient resources to benefit recovery. B. A nurse refuses to give information to a physician who is not responsible for the client's care. C. A nurse tricks a client into seclusion by asking the client to carry linen to the seclusion room. D. A nurse treats all of the clients equally regardless of illness severity.

ANS: C The nurse who tricks a client into seclusion has violated the ethical principle of veracity. The principle of veracity refers to one's duty to always be truthful and not intentionally deceive or mislead clients.

A client diagnosed with schizophrenia refuses to take medication, citing the right of autonomy. Under which circumstance would a nurse have the right to medicate the client against the client's wishes? A. When the client makes inappropriate sexual innuendos to a staff member B. When the client constantly demands inappropriate attention from the nurse C. When the client physically attacks another client after being confronted in group therapy D. When the client refuses to bathe or perform hygienic activities

ANS: C The nurse would have the right to medicate a client against his or her wishes if the client physically attacks another client. This client poses a significant risk to safety and is incapable of making rational choices. The client's refusal to accept treatment can be challenged because the client is endangering the safety of others.

Which is an accurate description of a common law? A. A common law would be invoked to deal with a nurse who, without justification, threatens a client with restraints. B. A common law would be invoked to deal with a nurse who touches a client without the client's consent. C. A common law would be invoked to deal with a hospital employee who steals drugs, hospital equipment, or both. D. A common law would be invoked to deal with a nurse's refusal to provide care for a specific client.

ANS: D Common laws apply to a body of principles that evolve from court decisions resolving various controversies. Common law may vary from state to state. Assault (threats) and battery (touch) are governed by civil law. Stealing is governed by criminal law.

A geriatric client is confused and wandering in and out of every door. Which scenario reflects the least restrictive alternative for this client? A. The client is placed in seclusion. B. The client is placed in a geriatric chair with tray. C. The client is placed in soft Posey restraints. D. The client is monitored by an ankle bracelet.

ANS: D The least restrictive alternative for this client would be monitoring by an ankle bracelet. The client does not pose a direct dangerous threat to self or others, so neither physical restraints nor seclusion would be justified.

An involuntarily committed client is verbally abusive to the staff and repeatedly threatening to sue. The client records the full names and phone numbers of the staff. Which nursing action is most appropriate to decrease the possibility of a lawsuit? A. Verbally redirect the client, and then limit one-on-one interaction. B. Involve the hospital's security division as soon as possible. C. Notify the client that documenting personal staff information is against hospital policy. D. Continue professional attempts to establish a positive working relationship with the client.

ANS: D The most appropriate nursing action is to continue professional attempts to establish a positive working relationship with the client. The involuntarily committed client should be respected and has the right to assert grievances if rights are infringed.

An inpatient psychiatric physician refuses to treat clients without insurance and prematurely discharges those whose insurance benefits have expired. Which violation of an ethical principle should a nurse recognize in this situation? A. Autonomy B. Beneficence C. Nonmaleficence D. Justice

ANS: D The nurse should determine that the ethical principle of justice has been violated by the physician's actions. The principle of justice requires that individuals should be treated equally regardless of race, sex, marital status, medical diagnosis, social standing, economic level, or religious belief.

Which statement should a nurse identify as correct regarding a client's right to refuse treatment? A. Clients can refuse pharmacological but not psychological treatment. B. Clients can refuse any treatment at any time. C. Clients can refuse only electroconvulsive therapy (ECT). D. Professionals can override treatment refusal if the client is actively suicidal or homicidal.

ANS: D The nurse should understand that health-care professionals can override treatment refusal when a client is actively suicidal or homicidal. A suicidal or homicidal client who refuses treatment may be a danger to self or others. This situation should be treated as an emergency, and treatment may be performed without informed consent.

1. Function independently

Advanced practice registered nurses generally: 1. Function independently 2. Function as unit directors 3. Work in acute care settings 4. Work in the university setting.

3. "Would you like for me to sit down with you for a few minutes so you can talk about this?"

After a health care provider has informed a patient that he has colon cancer, the nurse enters the room to find the patient gazing out the window in thought. The nurse's first response is which of the following? 1. "Don't be sad. People live with cancer every day." 2. "Have you thought about how you are going to tell your family?" 3. "Would you like for me to sit down with you for a few minutes so you can talk about this?" 4. "I know another patient whose colon cancer was cured by surgery."

3. Assessment

An 18-year-old woman is in the emergency department with fever and cough. The nurse obtains her vital signs, auscultates her lung sounds, listens to her heart sounds, determines her level of comfort, and collects blood and sputum samples for analysis. Which standard of practice is performed? 1. Diagnosis 2. Evaluation 3. Assessment 4. Implementation

1. Prepare for an influx of patients

At 3 am the emergency department nurse hears that a tornado hit the east side of town. What action does the nurse take first? 1. Prepare for an influx of patients 2. Contract the American Red Cross 3. Determine how to restore essential services 4. Evacuate patients per the disaster plan

Research on nurse retention has indicated that many nurses leave nursing after what period of time? Answers: A. before the end of the first year B. 5 years or less C. 7 years or less D. 10 years

B. 5 years or less Response Feedback: Hodges, Keely and Troyan note that many nurses leave the field after 5 years or less

1. Caregiver 2. Autonomy and accountability 3. Patient advocate 4. Health promotion

Contemporary nursing requires that the nurse has knowledge and skills for a variety of professional roles and responsibilities. Which of the following are examples? (Select all that apply.) 1. Caregiver 2. Autonomy and accountability 3. Patient advocate 4. Health promotion 5. Lobbyist

Legal BRN law requires hours of professional development every 2 years.

Correct 30

A nurse is doing some charting and needs a tool with help with legal protection. The nurse remembers that one useful tool is the acronym FLAT, which stands for: A. Factual, legible, accurate, and timely B. Foresight, legitimate, accountable, and timely C. Factual, legal, accurate, and time frame D. Fully accountable, legitimate, accurate and truthful

Correct A. Factual, legible, accurate, and timely

A nursing instructor is evaluating whether the nursing students understand the three fundamental qualities that leaders share. According to Bennis and Nanus, the fundamental qualities of effective leaders are: A. Guided vision, passion, and integrity B. Honesty, self-awareness, and sociability C. Knowledge of self, honesty, and maturity D. Intelligence, self-confidence, and determination

Correct A. Guided vision, passion, and integrity

A new graduate has been working on the medical-surgical unit for 3 months. When asked to take extra shifts or to rotate shifts, the nurse is more than willing. Lately the nurse has begun to wonder if he is sleep deprived at work. Some negative effects of sleep deprivation are listed below. Which of the following would the new graduate be least likely to experience if he is sleep deprived? A. A period of 24 hours of wakefulness is equivalent to a 0.001% blood alcohol level B. Increased risk of motor vehicle accidents C. Difficulty remembering and concentrating D. Increased risk of medical errors

CorrectA. A period of 24 hours of wakefulness is equivalent to a 0.001% blood alcohol level

As a student, you have learned that your duty as a nurse is to do good to others and maintain a balacne between those items that may cause harm and those that may cause good. Which ethical principle is this behavior based on? A. Beneficence B. Veracity C. Nonmaleficence D. Fidelity

CorrectA. Beneficence

A nurse is appointed to a leadership position in the local hospital. The nurse's position would be considered which of the following? A. Formal Leadership B. Informal leadership C. Leadership D. Management

CorrectA. Formal Leadership

Using a whiteboard to organize nursing assignments per shift or to provide patients information about their practitioner or diagnosis is a violation of which of the following? A. HIPAA B. Tort law C. Civil law D. OSHA

CorrectA. HIPAA

A nurse manager wishes to implement a risk management program because it: A. Has an emphasis upon quality improvement and protection from financial liability B. May investigate nurses who do not practice according to evidence-based practice C. Can be combined with quality maanagement goals; hence, there is no need for specially trained people D. Helps protect hosptials from bioethical problems

CorrectA. Has an emphasis upon quality improvement and protection from financial liability

At the end of the shift, nurses document the care provided each of the clients. The time spent on this type of accounting activity would be considered which of the following? A. Indirect care B. Nonproductive time C. Direct care D. Productive time

CorrectA. Indirect care

As a nurse you possess professional power. This type of power is that which: A. Is conferred on members of a profession B. Comes from one's position in a hierarchy C. Is perceived by a person with extensive knowledge D. Comes from a person's buying power or wealth

CorrectA. Is conferred on members of a profession

An expense that is associated with patient care is: A. Fixed B. Direct C. Variable D. Indirect

CorrectB. Direct

A nurse working in an immigrant community is aware that different racial, religious, and social groups have their own integrated patterns of human behavior that include language, thoughts, communication, action, values, and institutions. This is known as the group's: A. Values B. Tradition C. Culture D. Morals

CorrectC. Culture

Some elements of taxonomies based in the cognitive domain are listed here. Which does not necessarily belong? A. Comprehension B. Synthesis C. Knowledge D. Process

CorrectD. Process

You decide to conduct a study that begins with cigarette smoking and then looks at lung cancer. Which type of study will you conduct? A. Time series study B. Outcome study C. Retrospective study D. Prospective study

CorrectD. Prospective study

A nursing instructor explores with a group of students the range of different opportunities available for today's nurses. The instructor would recognize that clarification is needed if a student suggested which of the following? Answers: A. Pharmaceutical Sales B. Traveling Nurse C. Flight Nurse D. Reservations Nurse

D. Reservations Nurse

1. "I understand your reluctance, but the exercises are necessary for you to regain function in your shoulder. Let's go a bit more slowly and try to relax."

During a home health visit the nurse prepares to instruct a patient in how to perform range-of-motion (ROM) exercises for an injured shoulder. The nurse verifies that the patient took an analgesic 30 minutes before arrival at the patient's home. After discussing the purpose for the exercises and demonstrating each one, the nurse has the patient perform them. After two attempts with only the second of three exercises, the patient stops and says, "This hurts too much. I don't see why I have to do this so many times." The nurse applies the critical thinking attitude of integrity in which of the following actions? 1. "I understand your reluctance, but the exercises are necessary for you to regain function in your shoulder. Let's go a bit more slowly and try to relax." 2. "I see that you're uncomfortable. I'll call your doctor to decide the next step." 3. "Show me exactly where your pain is and rate it for me on a scale of 0 to 10." 4. "Is anything else bothering you? Other than the pain, is there any other reason you might not want to do the exercises?"

1. Completing incident reports when appropriate 2. Completing incident reports for a near miss 3. Communicating product concerns to an immediate supervisor

How does the nurse support a culture of society? (Select all that apply.) 1. Completing incident reports when appropriate 2. Completing incident reports for a near miss 3. Communicating product concerns to an immediate supervisor 4. Identifying the person responsible for an incident

1. Answer the call light promptly. 4. Answer questions honesty. 5. Demonstrate competence when doing treatments.

Identify behaviors that foster the development of trust. (Select all that apply.) 1. Answer the call light promptly. 2. Call the patient by first name unless requested otherwise. 3. Do all the care as quickly as possible and leave the room as the patient can rest. 4. Answer questions honesty. 5. Demonstrate competence when doing treatments.

3. Deflect your eyes downward to show respect

Mr. Sakda emigrated from Thailand. When taking care of him, you note that he looks relaxed and smiles but seldom looks at you directly. How do you respond? 1. Use therapeutic communication to assess for increased anxiety 2. Sit down and position yourself closer so you are at eye level 3. Deflect your eyes downward to show respect 4. Continue to maintain eye contact

3. In-service education

Nurses at a community hospital are in an education program to learn how to use a new pressure-relieving device for patients at risk for pressure ulcers. This is which type of education? 1. Continuing education 2. Graduate education 3. In-service education 4. Professional Registered Nurse Education

The six competencies of QSEN, as listed in the PP and the handout, (do in order) are:

Quality Improvement, Safety, Teamwork and Collaboration, Patient-centered care, Evidence-based practice, Informatics

List the 5 items to consider to do delegation corectly. Put them as listed as on PP slides.

Right Task Right Circumstance Right Person Right direction/communication Right Supervision/evaluation

3. knowledge application

The nurse asks a patient how she feels about her impending surgery for breast cancer. Before the discussion the nurse reviewed the description of loss and grief and therapeutic communication principles in his textbook. The critical thinking component involved in the nurse's review of the literature is: 1. experience 2. problem solving 3. knowledge application 4. clinical decision making

2. When the patient's pain medications are working 3. Just before lunch, when the patient is most awake and alert

The nurse is planning to teach a patient about the importance of exercise. When is the best time for teaching to occur? (Select all that apply.) 1. When there are visitors in the room 2. When the patient's pain medications are working 3. Just before lunch, when the patient is most awake and alert 4. When the patient is talking about current stressors in his or her life

4. The professional nurse works with colleagues and the patient's family to provide combined expertise in planning care.

The statement that best explains the role of collaboration with others for the patient's plan of care is which of the following? 1. The professional nurse consults the health care provider for direction in establishing goals for patients. 2. The professional nurse depends on the latest literature to complete an excellent plan of care for patients. 3. The professional nurse works independently to plan and deliver care and does not depend on other staff for assistance. 4. The professional nurse works with colleagues and the patient's family to provide combined expertise in planning care.

2. Analogy

When a nurse is teaching a patient about how to administer an epinephrine injection in case of a severe allergic reaction, he or she tells the patient to hold the injection like a dart. Which of the following instructional methods did the nurse use? 1. Telling 2. Analogy 3. Demonstration 4. Simulation

posttraumatic stress disorder (PTSD)

When doing an assessment of a young woman who was in an automobile accident 6 months before, the nurse learns that the woman has vivid images of the crash whenever she hears a loud, sudden noise. The nurse recognizes this as ___________________.

1. Results in neurophysiological response.

When teaching a patient about the negative feedback response to stress, the nurse indicates which of the following to describe the benefits of this stress response? 1. Results in neurophysiological response. 2. Reduces body temperature 3. Causes a person to be hypervigilant 4. Reduces level of consciousness to conserve energy.

3. Shifting quickly from subject to subject.

When working with an older adult, the nurse remembers to avoid: 1. Touching the patient. 2. Allowing the patient to reminisce. 3. Shifting quickly from subject to subject. 4. Asking the patient how he or she feels.

An essential characteristic of negotiation is ___. a. Win-lose b. Win-win c. Lose-lose d. None of the above.

a. Win-lose

Dashboards are ___. a. a combination of graphics and numbers to quickly display important data elements. b. a discrepancy between needs and resources. c. financial statements that include information about revenue sources. d. a way to chart medications given.

a. a combination of graphics and numbers to quickly display important data elements.

A balance sheet is ___. a. a financial statement that includes assets, liabilities, and equity. b. a financial statement that includes information about revenue sources and expenses at a specific point in time. c. a financial report that shows the cash inflow and outflow activities of financial stability of the organization.

a. a financial statement that includes assets, liabilities, and equity.

One of the problems with patient classification systems has been low validity. This problem includes the following: a. failing to account for the full scope of nursing practice. b. belief that patient classification systems are a vehicle to increase or decrease staffing levels inappropriately. c. wrongly projecting patient care needs for the next shift or time period. d. failing to use the data that is generated.

a. failing to account for the full scope of nursing practice.

All of the following are team fables except ____. a. team members are not always committed to implementing a good team process b. all team members are created equal c. team members generally use good critical thinking to resolve their questions and issues d. team members will set aside their emotional issues in the interest of effective team decision making

a. team members are not always committed to implementing a good team process

All of the following are team fables except ____. a. team members usually do not use good critical thinking to resolve their questions and issues b. each team member fully understands why they act the way they do c. people always want to work with others d. building trust is an important role of the team leader

a. team members usually do not use good critical thinking to resolve their questions and issues

Accountability is ___. a. the cornerstone of professional expression b. the same as responsibility c. to be avoided by professionals d. All of the above are true.

a. the cornerstone of professional expression

The following is not true about change ___. a. Learning how to thrive in the presence of continual change requires both art and science in the healthcare environment. b. All change is urgent. c. Not all change is value laden. d. Some good ideas just don't need to happen unless there is evidence and rationale for the change.

b. All change is urgent.

All of the following are essentials that evidence a good fit between supporting infrastructure and the capacity for sustainable professional practice except ___. a. Professionals organize around decisions, not positions. b. Decisions are never driven from the point of service. c. A grid of accountability demonstrates the locus of control for all decisions in the system and clearly identifies where decisions should be made and by whom. d. Clarification and distinction of decisions that are a priori management driven and owned and those that are staff owned is an important distinction of accountability in professional organizations.

b. Decisions are never driven from the point of service.

All of the following fall within the realm of complexity science except ___. a. behavioral economics b. linguistics c. gaming theory d. nonlinear dynamics

b. linguistics

<p> The law passed in 1996 which focused on the patient's right to confidentiality and the improvement of the portability and continuity of health insurance coverage is ___.</p> <p>a. the Good Samaritan Law</p> <p>b. the Health Insurance Portability and Accountability Act c. Informed Consent</p> <p>d. the Patient Self-Determination Act</p>

b. the Health Insurance Portability and Accountability Act

The obligation for social accountability represents the social contract between the society that licenses the profession and ___. a. those who practice a profession b. the persons who act in the best interests of the society c. employees of a healthcare institution d. None of the above.

b. the persons who act in the best interests of the society

All of the following are true except _____. a. Challenges that differences in personalities create need to be addressed. b. Values must have a voice, and that voice must be honored, respected, and included. c. All problems can be solved. d. Challenges that exist in matching demand to available resources need expression if conflict about them is to be allayed.

c. All problems can be solved.

The problems with prospective versus retrospective prediction of patient care needs includes___. a. the amount of staff and skill mix has been nearly impossible to determine without an acceptable error range without using a computerized solution. b. it is nearly impossible to accurately predict patient condition changes, new admissions, and discharges. c. a and b d. Neither is true.

c. a and b

Cash flow operating activities is ______. a. a financial statement that includes assets, liabilities, and equity. b. a financial statement that includes information about revenue sources and expenses at a specific point in time. c. a financial report that shows the cash inflow and outflow activities of financial stability of the organization.

c. a financial report that shows the cash inflow and outflow activities of financial stability of the organization.

An American school of philosophy that rejects the esoteric metaphysics of traditional European academic philosophy in favor of more down-to-earth, concrete questions and answers is ____. a. morality b. utilitarianism c. pragmatism d. bioethics

c. pragmatism

Inappropriate communication with patients could involve all of the following ethical issues except ____. a. autonomy b. betrayal c. pragmatism d. nonmaleficense

c. pragmatism

All of the following are examples situations that could lead to nurse-patient ethical dilemmas except ____. a. pain management b. patient restraint c. patient privacy d. All of the above are true.

d. All of the above are true.

Individuals at the point of service will need to be ___. a. more self-directed b. more collaborative c. more reliant on management d. a and b

d. a and b

Job descriptions ___. a. have been used historically as a vehicle for delineating role functions, tasks, and activities of the nurse. b. do not always match the dynamic nature of the work of nursing. c. are written so that they fit all possibilities in the work environment. d. a and b e. All of the above are true.

d. a and b

<p>Collective bargaining agreements include all of the following except ___.</p> <p>a. salaries</p> <p>b. benefits</p> <p>c. working conditions</p> <p>d. health care costs</p>

d. health care costs

The following elements need to be included **********ignment of work ___. a. individuals charged with responsibility for a particular goal need to be clear about the specifics and the expectations. b. members working on an activity need to be clear about the time and resources needed to undertake the work. c. the work timetable is generally not as important as the resource utilization. d. measures of success must be incorporated into the team member work. e. A, b, and d are correct. f. All of the above are correct.

e. A, b, and d are correct.

The following is true about goals _____. a. they give the team clear and specific direction b. they create a point of reference where teams can evaluate their progress and determine movement toward completion c. they are generally frowned upon by hospital administration d. they involve both the team and individual members in defining the performance expectation of the team e. A, b, and d are correct. f. All of the above are correct.

e. A, b, and d are correct.

Strategies to enhance personal balance include ____. a. minimizing negative thoughts b. specific time for reflection and planning for the future c. being mindful of self within or after professional practice situations d. stress management e. All of the above are helpful.

e. All of the above are helpful.

For a system to be effective the following considerations must be addressed ___. a. Complex adaptive system providers are integrated in a collaborative and linked relationship that synthesizes well around the needs of the user. b. The primary driving point of decision making in a complex clinical delivery system is the place where the provider and the user meet. c. Data must be available at point-of-service to support caregivers in decision making, in clinical activities, and in evaluation of impact and value. d. a and b e. All of the above are true.

e. All of the above are true.

The following are important considerations in constructing the basic core schedule: a. anticipated patient needs volume. b. shift length. c. experience. d. available research evidence for staffing effectiveness. e. All of the above are true.

e. All of the above are true.

The role of advanced practice nurse providers and clinical experts includes___. a. writing orders. b. providing general patient care oversight. c. providing care. d. assisting staff in the care of more complex patients using the latest evidence in a cost-effective approach. e. All of the above are true.

e. All of the above are true.

<p>The following are strategies to support effective delegation ___.</p> <p>a. Clearly identify the importance of requesting assistance when needed.</p> <p>b. Gain an understanding of personal delegation competence.</p> <p>c. Provide feedback on tasks according to the guidelines specified by the delegator.</p> <p>d. Access evidence-based resources for delegation and supervision.</p> <p>e. b and d</p>

e. b and d

All of the following are elements of readying for negotiation except ___. a. understanding the issues at the table b. determining the personal bottom line c. understanding the bottom line of each participant d. practicing the approach to the interaction e. determining how to defeat the opponent

e. determining how to defeat the opponent

5. Patient care needs categories include___. a. cognitive needs. b. pain and comfort needs. c. family information and support needs. d. treatments and interventions. e. a and c f. All of the above are true.

f. All of the above are true.

The debate related to nursing as a profession relates to ___. a. the predominant level of minimum education of the majority of the profession b. its dependency role in relationship to other disciplines c. its lack of clarity with regard to its specific contribution to the health of society d. an ongoing lack of a disciplined profession e. a and c f. All of the above are true.

f. All of the above are true.

The role of management in shared governance is ___. a. stewardship b. unilateral decision making c. servant leadership d. assure seamless interface between the system's purpose and resources e. a and c f. a, c, and d

f. a, c, and d

Elements essential for viable team construction are___. a. purpose b. goals c. roles d. relationships e. activities and functions f. a, b, and d g. All of the above are true.

g. All of the above are true.

The next stage of conflict is seen with _____. a. verbalizations b. snide remarks c. statements of discomfort d. cynicism e. negative assessments f. b, c, and d are true. g. All of the above are true.

g. All of the above are true.

Supply of healthcare goods and services includes ___. a. people b. supplies c. technology d. time f. funds to pay for services g. All of the above.

g. All of the above.

SBAR stands for S______________B________________A_______________R______________________

Situation, Background, Assessment, Recommendation

4. Feedback

The nurse summarizes the conversation with the patient to determine if the patient has understood him or her. This is what element of the communication process? 1. Referent 2. Channel 3. Environment 4. Feedback

Decision making ___. a. is a complex cognitive process. b. involves choosing a particular course of actions from among alternatives. c. is an essential component of the problem-solving process. d. All of the above are true.

d. All of the above are true.

3, 5, 2, 4, 1

A 67-year old patient will be discharged from the hospital in the morning. The health care provider has ordered three new medications for her. Place the following steps of the nursing process in the correct order: 1. The nurse returns to the patient's room and asks her to describe the medications she will be taking at home. 2. The nurse talks with the patient and family about who will be available if the patient has difficulty taking medicines and considers consulting with the health care provider about a home health visit. 3. The nurse asks the patient if she is in pain, feels tired, and is willing to spend the next few minutes learning about her medications. 4. The nurse brings the containers of medicines and information leaflets to the bedside and discusses each medication with her. 5. The nurse considers what she learns from the patient and identifies the patient's nursing diagnosis.

2. After 6 weeks when she adjusts to the child's respiratory status and reestablishes the entitlement checks

A crisis intervention nurse working with a mother whose Down syndrome child has been hospitalized with pneumonia and who has lost her entitlement check while the child is hospitalized can expect the mother to regain the stability after how long? 1. After 2 weeks when the child's pneumonia begins to improve 2. After 6 weeks when she adjusts to the child's respiratory status and reestablishes the entitlement checks 3. After 1 month when the child goes home and the mother gets help from a food pantry 4. After 6 months when the child is back in school

4. Informatics

A critical care nurse is using a computerized decision support system to correctly position her ventilated patients to reduce pneumonia cause by accumulated respiratory secretions. This is an example of which Quality and Safety in the Education of Nurses (QSEN) competency? 1. Patient-centered care 2. Safety 3. Teamwork and collaboration 4. Informatics

3. Analyticity 4. Self-Confidence

A nurse has worked on an oncology unit for 3 years. One patient has become visibly weaker and states. "I feel funny." The nurse knows how patients often have behavior changes before developing sepsis when they have cancer. The nurse asks the patient questions to assess thinking skills and notices the patient shivering. The nurse goes to the phone, calls the physician, and begins the conversation by saying, "I believe that your patient is developing sepsis. I want to report symptoms I'm seeing." What examples of critical thinking concepts does the nurse show? (Select all that apply.) 1. Experience 2. Ethical 3. Analyticity 4. Self-Confidence 5. Risk taking

2. Advocate

A nurse is caring for a patient with end-stage lung disease. The patient wants to go home on oxygen and be comfortable. The family wants the patient to have a new surgical procedure. The nurse explains the risk and benefits of the surgery to the family and discusses the patient's wishes with the family. The nurse is acting as the patient's: 1. Educator 2. Advocate 3. Caregiver 4. Case manager

2. Safety

A nurse is caring for an older-adult couple in a community-based assisted living facility. During the family assessment he notes that the couple has many expired medications and multiple medications for their respective chronic illnesses. They note that they go to two different health care providers. The nurse begins to work with the couple to determine what they know about their medications and helps them decide on one care provider rather than two. This is an example of which Quality and Safety in the Education of Nurses (QSEN) competency? 1. Patient-centered care 2. Safety 3. Teamwork 4. Informatics

3. The patient will perform breast self-examination correctly on herself before the end of the teaching session.

A nurse is going to teach a patient how to perform breast self-examination. Which behavioral objective does the nurse set to best measure the patient's ability to perform the examination? 1. The patient will verbalize the steps involved in breast self-examination within 1 week. 2. The nurse will explain the importance of performing breast self-examination once a month. 3. The patient will perform breast self-examination correctly on herself before the end of the teaching session. 4. The nurse will demonstrate breast self-examination on a breast model provided by American Cancer Society.

2. A learning objective

A nurse is planning a teaching session about healthy nutrition with a group of children who are in first grade. The nurse determines that after the teaching session the children will be able to name three examples of foods that are fruits. This is an example of: 1. A teaching plan 2. A learning objective 3. Reinforcement of content 4. Enhancing the children's self-efficacy

4. Health promotion and illness prevention

A nurse is teaching a group of young college-age women the importance of using sunscreen when going out in the sun. What type of content is the nurse providing? 1. Simulation 2. Restoring health 3. Coping with impaired function 4. Health promotion and illness prevention

1. A nurse explain to the NAP the approach to use in getting the patient up and why the patient has activity limitations. 3. The nurse sees the NAP preparing to help a patient out of bed, goes to assist, and thanks the NAP for her efforts to get the patient up early. 4. The nurse is in Patient B's room to check an intravenous (IV) line and collects the urine specimen while in the room.

A nurse is working with a nursing assistive personnel (NAP) on a bust oncology unit. The nurse has instructed the NAP on the tasks that need to be performed, including getting patient A out of bed, collecting a urine sample from patient B, and checking vital signs on patient C, who is scheduled to go home. Which of the following represent(s) successful delegation? (Select all that apply.) 1. A nurse explain to the NAP the approach to use in getting the patient up and why the patient has activity limitations. 2. A nurse is asked by a patient to help her to the bathroom; the nurse leaves the room and directs the NAP to assist the patient instead. 3. The nurse sees the NAP preparing to help a patient out of bed, goes to assist, and thanks the NAP for her efforts to get the patient up early. 4. The nurse is in Patient B's room to check an intravenous (IV) line and collects the urine specimen while in the room. 5. The nurse offers to support the NAP when needed but allows her to complete patient care tasks without constant oversight.

1. How to use an inhaler during an asthma attack

A nurse needs to teach a young woman newly diagnosed with asthma how to manage her disease. Which of the following topics does the nurse teach first? 1. How to use an inhaler during an asthma attack 2. The need to avoid people who smoke to prevent asthma attacks 3. Where to purchase a medical alert bracelet that says she has asthma 4. The importance of maintaining a healthy diet and exercising regularly

3. Chronic illness

A nurse observes that a patient whose home life is chaotic with intermittent homelessness, a child with spina bifida, and an abusive spouse appears to be experiencing an allostatic load. As a result, the nurse expects to detect which of the following while assessing the patient? 1. Posttraumatic stress disorder 2. Rising hormone levels 3. Chronic illness 4. Return of vital signs to normal

2. Think about past experience with patients who develop postoperative complications. 3. Decide which activities can be combined for patient B and C.

A nurse who is working on a surgical unit is caring for four different patients. Patient A will be discharged home and is in need of instruction about wound care. Patients B and C have returned from the operating room within an hour of each other, and both require vital signs and monitoring of their intravenous (IV) lines. Patient D is resting following a visit by physical therapy. Which of the following activities by the nurse represent(s) use of clinical decision making for groups of patients? (Select all that apply.) 1. Consider how to involve patient A in deciding whether to involve the family caregiver in wound care instruction. 2. Think about past experience with patients who develop postoperative complications. 3. Decide which activities can be combined for patient B and C. 4. Carefully gather any assessment information and identify patient problems.

2. Evaluation

A patient had hip surgery 16 hours ago. During the previous shift the patient had 40 mL of drainage in the surgical drainage collection device for an 8 hour period. The nurse refers to the written plan of care, noting that the health care provider is to be notified when drainage in the device exceeds 100 mL for the day. On entering the room, the nurse looks at the device and carefully notes the amount of drainage currently in it. This is an example of: 1. Planning 2. Evaluation 3. Intervention 4. Diagnosis

2. Expressing the importance of learning the skill correctly

A patient needs to learn how to administer a subcutaneous injection. Which of the following reflects that the patient is ready to learn? 1. Describing difficulties a family member has had in taking insulin 2. Expressing the importance of learning the skill correctly 3. Being able to see and understand the markings on the syringe 4. Having the dexterity needed to prepare and inject the medication

4. Psychomotor domain

A patient needs to learn to use a walker. Which domain is required for learning this skill? 1. Affective domain 2. Cognitive domain 3. Attentional domain 4. Psychomotor domain

3. Conducting reflective practice.

A patient on a surgical unit develops sudden shortness of breath and a drop in blood pressure. The staff respond, but the patient dies 30 minutes later. The manager on the nursing unit calls the staff involved in the emergency response together. The staff discusses what occurred over the 30-minute time frame, the actions taken, and whether other steps should have been implemented. The nurses in this situation are: 1. Problem solving. 2. Showing humility. 3. Conducting reflective practice. 4. Exercising responsibility.

1. Denial.

A patient who is having difficulty managing his diabetes mellitus responds to the news that his hemoglobin A1C, a measure of blood sugar control over the past 90 days, has increased by saying, "the hemoglobin A1C is wrong. My blood sugar levels have been excellent for the last 6 months." The patient is using the defense mechanism: 1. Denial. 2. Conversion. 3. Dissociation. 4. Displacement.

2. Demonstration

A patient who is hospitalized has just been diagnosed with diabetes. He is going to need to learn how to give himself injections. Which teaching method does the nurse use? 1. Simulation 2. Demonstration 3. Group instruction 4. One-on-one discussion

1. Telling approach

A patient with chest pain is having an emergency cardiac catheterization. Which teaching approach does the nurse use in this situation? 1. Telling approach 2. Selling approach 3. Entrusting approach 4. Participating approach

3, 4, 1, 5, 2

A physician writes an order to apply a wrist restraint to a patient who has been pulling out a surgical wound drain. Place the following steps for applying the restraint in the correct order. 1. Explain what you plan to do. 2. Wrap a limb restraint around wrist or ankle with soft part toward skin and secure. 3. Determine that restraint alternatives fail to ensure patient's safety. 4. Identify the patient using proper identifier. 5. Pad the patient's wrist.

You have been identified as a Millennial Generation nurse. You are working with a team of three LVNs , two of whom are Baby Boomers and one is a Generation X nurse. You also are working with two CNAs both who are from the Baby Boomers Generation. What should you focus on when working with your team? Answers: A. Communication and teamwork among staff of all generations is critical in assuring a positive work milieu that can affect safe patient care and good working relationships B. All of the staff members will resent you because you are much younger and more educated than any of them C. Your reputation is dependent on whether the team members acknowledge your authority and provide you with respect even if you are the youngest person on the team D. Providing the LVNs with additional responsibilities and have then supervise the CNAs in what ever manner they choose

A. Communication and teamwork among staff of all generations is critical in assuring a positive work milieu that can affect safe patient care and good working relationships

A nursing instructor is evaluating a student's understanding of the primary difference between QA and QI. Which response by the student would indicate that the student understood? A. QA's emphasis is on maintaining minimum standards of care, and QI's emphasis is upon identifying real and potential problems B. QA documents quality, and QI reports incidents and errors C. QI is more a single program, and QA is more a management approach D. QI is reactive and QA is proactive

A. QA's emphasis is on maintaining minimum standards of care, and QI's emphasis is upon identifying real and potential problems

A career counselor is meeting with a group of nursing graduates to discuss the preparation of their resumes. During the discussion the counselor asks the students which elements are important to include in a quality resume. Which element suggested by a student would indicate to the counselor that further clarification is needed? Answers: A. Humorous references are always appreciated B. Off-white or ivory high quality paper with matching envelopes C. Accurate spelling and grammar D. Accurate dates

A. Humorous references are always appreciated

what makes an occupation a profession> Listed below are some of the characteristics of professions. Which is not necessarily correct? Answers: A. The members are self organizing and establish standards for education through a small group of experts. B. Body of specialized knowledge is continually developed and evaluated by research. C. It is an occupation that involves a unique practice that carries individual responsibility and is based on theoretical knowledge. D. Members continually evaluate the quality of services provided to protect individuals and the public.

A. The members are self organizing and establish standards for education through a small group of experts.

An inpatient client, whom the treatment team has determined to be a danger to self, gives notice of intention to leave the hospital. What information should the nurse recognize as having an impact on the treatment team's next action? A. State law determines how long a psychiatric facility can hold a client. B. Federal law determines if the client is competent. C. The client's family involvement will determine if discharge is possible. D. Hospital policies will determine treatment team actions.

ANS: A Most states commonly cite that in an emergency a client who is dangerous to self or others may be involuntarily hospitalized.

Without authorization, a nurse administers an extra dose of narcotic tranquilizer to an agitated client. The nurse's coworker observes this action but does nothing for fear of repercussion. What is the ethical interpretation of the coworker's lack of involvement? A. Taking no action is still considered an unethical action by the coworker. B. Taking no action releases the coworker from ethical responsibility. C. Taking no action is advised when potential adverse consequences are foreseen. D. Taking no action is acceptable because the coworker is only a bystander.

ANS: A The coworker's lack of involvement can be interpreted as an unethical action. The coworker is experiencing an ethical dilemma in which a decision needs to be made between two unfavorable alternatives. The coworker has a responsibility to report any observed unethical actions.

What is the legal significance of a nurse's action when the nurse threatens a demanding client with restraints? A. The nurse can be charged with assault. B. The nurse can be charged with negligence. C. The nurse can be charged with malpractice. D. The nurse can be charged with beneficence.

ANS: A Assault is an act that results in a person's genuine fear and apprehension that he or she will be touched without consent.

A client diagnosed with schizophrenia receives fluphenazine decanoate (Prolixin Decanoate) from a home health nurse. The client refuses medication at one regularly scheduled home visit. Which nursing intervention is ethically appropriate? A. Allow the client to decline the medication and document. B. Tell the client that if the medication is refused, hospitalization will occur. C. Arrange with a relative to add medication to the client's morning orange juice. D. Call for help to hold the client down while the injection is administered.

ANS: A It is ethically appropriate for the nurse to allow the client to decline the medication and provide accurate documentation. The client's right to refuse treatment should be upheld unless the refusal puts the client or others in harm's way.

A nurse should recognize that clients who have a history of missed or late medical appointments are most likely to come from which cultural group? A. African Americans B. Asian Americans C. Native Americans D. Jewish Americans

ANS: C The nurse should recognize that Native American clients might have a history of missed or late medical appointments. Many Native Americans are not ruled by the clock. The concept of time is casual and focused on the present.

A psychiatric nurse working on an inpatient unit receives a call asking if an individual has been a client in the facility. Which nursing response reflects appropriate legal and ethical obligations? A. Refusing to give any information to the caller, citing rules of confidentiality B. Refusing to give any information to the caller by hanging up C. Affirming that the person has been seen at the facility but providing no further information D. Suggesting that the caller speak to the client's therapist

ANS: A The most appropriate action by the nurse is to refuse to give any information to the caller. Admission to the facility would be considered protected health information (PHI) and should not be disclosed by the nurse without prior client consent.

A Native American client is admitted to an emergency department (ED) with an ulcerated toe secondary to uncontrolled diabetes mellitus. The client refuses to talk to a physician unless a shaman is present. Which nursing intervention is most appropriate? A. Try to locate a shaman that will agree to come to the ED. B. Explain to the client that "voodoo" medicine will not heal the ulcerated toe. C. Ask the client to explain what the shaman can do that the physician cannot. D. Inform the client that refusing treatment is a client's right.

ANS: A The most appropriate nursing intervention would be to try to locate a shaman who will agree to come to the ED. The nurse should understand that in the Native American culture, religion and health-care practices are often intertwined. The shaman, a medicine man, may confer with physicians regarding the care of a client. Research supports the importance of both health-care systems in the overall wellness of Native American clients.

In what probable way should a nurse expect an Asian American client to view mental illness? A. Mental illness relates to uncontrolled behaviors that bring shame to the family. B. Mental illness is a curse from God related to immoral behaviors. C. Mental illness is cured by home remedies based on superstitions. D. Mental illness is cured by "hot and cold" herbal remedies.

ANS: A The nurse should expect that many Asian Americans are most likely to view mental illness as uncontrolled behavior that brings shame to the family. It is often more acceptable for mental distress to be expressed as physical ailments.

A client requests information on several medications in order to make an informed choice about management of depression. A nurse should provide this information to facilitate which ethical principle? A. Autonomy B. Beneficence C. Nonmaleficence D. Justice

ANS: A The nurse should provide the information to support the client's autonomy. A client who is capable of making independent choices should be permitted to do so. In instances when clients are incapable of making informed decisions, a legal guardian or representative would be asked to give consent.

After disturbing the peace, an aggressive, disoriented, unkempt, homeless individual is escorted to an emergency department by police. The client threatens suicide. Which criteria would enable a physician to consider involuntary commitment? (Select all that apply.) A. Being dangerous to others B. Being homeless C. Being disruptive to the community D. Being gravely disabled and unable to meet basic needs E. Being suicidal

ANS: A, D, E The physician could consider involuntary commitment when a client is being dangerous to others, is gravely disabled, or is suicidal. If the client is determined to be mentally incompetent, consent should be obtained from the legal guardian or court-approved guardian or conservator. A hospital administrator may give permission for involuntary commitment when time does not permit court intervention.

Which is an example of an intentional tort? A. A nurse fails to assess a client's obvious symptoms of neuroleptic malignant syndrome. B. A nurse physically places an irritating client in four-point restraints. C. A nurse makes a medication error and does not report the incident. D. A nurse gives patient information to an unauthorized person.

ANS: B A tort is a violation of civil law in which an individual has been wronged and can be intentional or unintentional. A nurse who physically places an irritating client in restraints has touched the client without consent and has committed an intentional tort.

A nursing instructor is teaching about cultural characteristics. Which statement by the student indicates the need for further instruction? A. "All cultures communicate freely within their group." B. "All cultures embrace light therapeutic touch." C. "All cultures view the importance of timeliness differently." D. "All cultures display biological variations."

ANS: B All cultures do not embrace light therapeutic touch. In the Native American culture, if a hand is offered to another it may be accepted with a light touch; however, in the Asian culture, touching during communication has been historically considered unacceptable. This student statement indicates the need for further instruction.

In the situation presented, which nursing intervention constitutes false imprisonment? A. The client is combative and will not redirect stating, "No one can stop me from leaving." The nurse seeks the physician's order after the client is restrained. B. The client has been consistently seeking the attention of the nurse much of the day. The nurse institutes seclusion. C. A psychotic client, admitted in an involuntary status, runs off the psychiatric unit. The nurse runs after the client and the client agrees to return. D. A client hospitalized as an involuntary admission attempts to leave the unit. The nurse calls the security team and they prevent the client from leaving.

ANS: B False imprisonment is the deliberate and unauthorized commitment of a person within fixed limits by the use of verbal or physical means. Seclusion should only be used in an emergency situation to prevent harm after least restrictive means have been unsuccessfully attempted.

In response to a student's question regarding choosing a psychiatric specialty, a charge nurse states, "Mentally ill clients need special care. If I were in that position, I'd want a caring nurse also." From which ethical framework is the charge nurse operating? A. Kantianism B. Christian ethics C. Ethical egoism D. Utilitarianism

ANS: B The charge nurse is operating from a Christian ethics framework. The imperative demand of Christian ethics is to treat others as moral equals by permitting them to act as we do when they occupy a position similar to ours. Kantianism states that decisions should be made based on moral law and that actions are bound by a sense of moral duty. Utilitarianism holds that decisions should be made focusing on the end result being happiness. Ethical egoism promotes the idea that what is right is good for the individual.

Which situation exemplifies both assault and battery? A. The nurse becomes angry, calls the client offensive names, and withholds treatment. B. The nurse threatens to "tie down" the client and then does so against the client's wishes. C. The nurse hides the client's clothes and medicates the client to prevent elopement. D. The nurse restrains the client without just cause and communicates this to family.

ANS: B The nurse in this situation has committed both assault and battery. Assault refers to an action that results in fear and apprehension that the person will be touched without consent. Battery is the touching of another person without consent.

When working with clients of a particular culture, which action should a nurse avoid? A. Maintaining eye contact based on cultural norms B. Assuming that all individuals who share a culture or ethnic group are similar C. Supporting the client in participating in cultural and spiritual rituals D. Using an interpreter to clarify communication

ANS: B The nurse should avoid assuming that all individuals who share a culture or ethnic group are similar. This action constitutes stereotyping and must be avoided. Within each culture, many variations and subcultures exist. Clients should be treated as individuals.

An African American youth, growing up in an impoverished neighborhood, seeks affiliation with a black gang. Soon he is engaging in theft and assault. What cultural consideration should a nurse identify as playing a role in this youth's choices? A. Most African American homes are headed by strong, dominant father figures. B. Most African Americans choose to remain within their own social organization. C. Most African Americans are uncomfortable expressing emotions and need group affiliations. D. Most African Americans have limited religious beliefs which contribute to criminal activity.

ANS: B The nurse should identify that a tendency to remain within one's own social organization may have played a role in this youth's choice to join a black gang. African Americans who have assimilated into the dominant culture are likely to be well educated and future focused. Those who have not assimilated may be unemployed or have low-paying jobs, and view the future as hopeless given their previous encounters with racism and discrimination.

Which client should a nurse identify as a potential candidate for involuntary commitment? A. A client living under a bridge in a cardboard box B. A client threatening to commit suicide C. A client who never bathes and wears a wool hat in the summer D. A client who eats waste out of a garbage can

ANS: B The nurse should identify the client threatening to commit suicide as eligible for involuntary commitment. The suicidal client who refuses treatment is a danger to self and requires emergency treatment.

Because of cultural characteristics, in which of the following cultural groups would a nurse's assessment of mood and affect be most challenging? (Select all that apply.) A. Arab Americans B. Native Americans C. Latino Americans D. Western European Americans E. Asian Americans

ANS: B, E The nurse should expect that both Native Americans and Asian Americans might be difficult to assess for mood and affect. In both cultures, expressing emotions is difficult. Native Americans are encouraged to not communicate private thoughts. Asian Americans may have a reserved public demeanor and may be perceived as shy or uninterested.

The nursing staff is discussing the concept of competency. Which information about competency should a nurse recognize as true? A. Competency is determined with a client's compliance with treatment. B. Refusal of medication can initiate an incompetency hearing leading to forced medications. C. A competent client has the ability to make reasonable judgments and decisions. D. Competency is a medical determination made by the client's physician.

ANS: C A competent individual's cognition is not impaired to an extent that would interfere with decision making.

Which cultural group is correctly matched with the disease process for which this group is most susceptible? A. African Americans are susceptible to lactose intolerance. B. Western European Americans are susceptible to malaria. C. Arab Americans are susceptible to sickle cell disease. D. Jewish Americans are susceptible to thalassemia.

ANS: C A number of genetic diseases are more common in the Arab American population, including sickle cell disease, tuberculosis, malaria, trachoma, typhus, hepatitis, typhoid fever, dysentery, parasitic infestations, thalassemia, and cardiovascular disease.

A client is concerned that information given to the nurse remains confidential. Which is the nurse's best response? A. "Your information is confidential. It will be kept just between you and I." B. "I will share the information with staff members only with your approval." C. "If the information impacts your care, I will need to share it with the treatment team." D. "You can make the decision whether your physician needs this information or not."

ANS: C Basic to the psychiatric client's hospitalization is his or her right to confidentiality and privacy. When admitted to an inpatient psychiatric facility, a client gives implied consent for information to be shared with health-care workers specifically involved in the client's care.

A Latin American woman refuses to participate in an assertiveness training group. Which cultural belief should a nurse identify as most likely to have influenced this client's decision? A. Future orientation causes the client to devalue assertiveness skills. B. Decreased emotional expression makes it difficult to be assertive. C. Assertiveness techniques may not be aligned with the client's definition of the female role. D. Religious prohibitions prevent the client's participation in assertiveness training.

ANS: C The nurse should identify that the Latin American woman's refusal to participate in an assertiveness training group may be influenced by the Latin American cultural definition of the female role. Latin Americans place a high value on the family which is male dominated. The father usually possesses the ultimate authority.

A client who will be receiving electroconvulsive therapy (ECT) must provide informed consent. Which situation should cause a nurse to question the validity of the informed consent? A. The client is paranoid. B. The client is 87 years old. C. The client incorrectly reports his or her spouse's name, date, and time of day. D. The client relies on his or her spouse to interpret the information.

ANS: C The nurse should question the validity of informed consent when the client incorrectly reports the spouse's name, date, and time of day. This indicates that this client is disoriented and may not be competent to make informed choices.

When planning client care for a Latino American, the nurse should be aware of which cultural influence that may impact access to health care? A. The root doctor may be the first contact made when illness is encountered. B. The "yin" and "yang" practitioner may be the first contact made when illness is encountered. C. The shaman may be the first contact made when illness is encountered. D. The curandero may be the first contact made when illness is encountered.

ANS: D The nurse should understand that some Latin Americans may initially contact a curandero when illness is encountered. The curandero is the folk healer who is believed to have a gift from God for healing the sick. Treatments often include supernatural rituals, prayers, magic, practical advice, and indigenous herbs.

Which rationale by a nursing instructor best explains why it is challenging to globally classify the Asian American culture? A. Extremes of emotional expression prevent accurate assessment of this culture. B. Suspicion of Western civilization has resulted in minimal cultural research. C. The small size of this subpopulation makes research virtually impossible. D. The Asian American culture includes individuals from many different countries.

ANS: D The nursing instructor's best explanation is that the Asian American culture is difficult to classify globally due to the number of countries that identify with this culture. The Asian American culture includes peoples and descendents from Japan, China, Vietnam, the Philippines, Thailand, Cambodia, Korea, Laos, India, and the Pacific Islands. Within this culture there are vast differences in values, religious practices, languages, and attitudes.

4. Allows the patient time to express himself or herself and ask questions

And older adult is being started on a new antihypertensive medication. In teaching the patient about the medication, the nurse: 1. Speaks loudly 2. Presents the information once 3. Expects the patient to understand the information quickly 4. Allows the patient time to express himself or herself and ask questions

In situational theory, a telling leadership style is considered: A. High task, high relationship behavior B. High task, low relationship behavior C. Low task, low relationship behavior D. Low task, high relationship behavior

B. High task, low relationship behavior

The field of advanced practice nursing (APN) has grown since its inception in the 1800s. In this list of types of APN degrees, which is not necessarily correct? Answers: A. CRNA (Certified Registered Nurse Anesthetist) B. CRN (Certified Registered Nurse) C. CNM (Certified Nurse Midwife) D. NP (Nurse Practitioner)

B. CRN (Certified Registered Nurse)

The employer-applicant interview contains several phases. Which of the following is not necessarily one of these phases? Answers: A. Preparation B. Evaluation phase C. Working phase D. Introductory phase

B. Evaluation phase

An ongoing issue in nursing has been the minimum appropriate entry-level requirements to practice as a registered nurse and what programs are qualified to educate professional nurses for the RN degree. As a result numerous motions and suggestions have been made regarding the RN degree. Which of the following is NOT a part of these contributing factors? Answers: A. Two-year associate degree and three-year diploma schools can grant a RN degree B. Only four-year baccalaureate degree school can grant viable RN degrees C. 1965 American Nurses Association (ANA) House of Delegates motion to work toward a baccalaureate as the foundation for professional nursing practice D. 1985 ANA House of Delegates agreed to urge state nursing associations to establish the baccalaureate degree as the minimum for RNs

B. Only four-year baccalaureate degree school can grant viable RN degrees

In class, students tell the teacher that they often watch or observe themselves as they perform tasks or make decisions about particular situations. The teacher would most likely inform the students that what they are doing is called: A. Reflective thinking B. Critical thinking C. Intuitive thinking D. Decision making

Correct A. Reflective thinking

When a nurse considers delegating to a CNA, what Five Rights should be utilized? A. Right patient, right chart, right doctor, right results, right information B. Right person, right patient, right task, right time frame, right side C. Right task, right circumstance, right person, right direction/communication, right supervision D. Right room, right time, right person, right documentation, right nurse

Correct C. Right task, right circumstance, right person, right direction/communication, right supervision

A nurse manager meets regularly with other nurse managers, participates on the organization's committees, and attends meetings sponsored by professional organizations in order to manage relationships. These activities are considered which function of a manager? A. Monitoring B. Problem solving C. Informing D. Networking

Correct D. Networking

Legitimate power happens when people have information others must have True or False

Correct False Response Feedback: Legitimate power is related to position not knowledge

The first paragraph of your cover letter is to share the qualities you bring to the job True or False

Correct False Response Feedback: The first paragraph is to praise the employer and share why you want to work for them

Followship is a passive role and requires nothing of the employee. True or False

Correct False Response Feedback: followship is an active role and requires action of the part of an employee

In the BRN Scope of Practice for the RN includes the nursing process in the role of the RN True or False

Correct True

In an effort to provide evidence-based care, the nurse manager of the unit informs the staff that they will be benchmarking thier wound care practices with that of the major medical center in the area. The staff recognizes that they will be doing which of the following? A. Measuring their wound care practices against one of their hospital's toughest competitors, a recognized leading hospital in the area B. Working harder for longer hours each shift C. Comparing the number of cases of wound care in their hospital with the number of cases in the other hospital D. Competing with another hospital to determine who provides the better care

CorrectA. Measuring their wound care practices against one of their hospital's toughest competitors, a recognized leading hospital in the area

An instructor wants to determine whether a nursing student knows during which era the cost of health care began to be questioned. Which response by the student would indicate that the student knows? A. 1960s B. 1990s C. 1940s D. 1950s

CorrectA. 1960s

The Pareto Principle pertains to a strategy for balancing life and work through the prioritization of efforts. Which is correct regarding this principle? A. 80% of unfocused effort results in 20% of results B. 30% of unfocused effort results in 70% of results C. 20% of focused efforts results in 20% of results D. 80% of focused effort results in 20% of results

CorrectA. 80% of unfocused effort results in 20% of results

The act of being responsible for the actions or inactions of yourself and of others in nursing is: A. Accountability B. Assignment C. Delegation D. Authority

CorrectA. Accountability

A nurse preceptor wants to determine if a novice nurse is able to organize tasks and categorize then according to patient needs and conditions. Which of the following categories, if included by the novice, would indicate to the preceptor that further teaching is needed? A. Activities essential to hospital/governmental regulation B. Activities essential to the plan of care C. Life-threatening or potentially life-threatening conditions D. Activities essential to patient safety

CorrectA. Activities essential to hospital/governmental regulation

The nurse is caring for an 84-year-old male client diagnosed with benign prostatic hypertrophy. The client has undergone a transurethral resection of the prostte (TURP) and is complaining of bladder spasms. Which intervention should the nurse implement first? A. Assess the client's three-way urinary catheter for patency B. Calculate the client's urinary output C. Administer an antispasmodic medication for bladder spasms D. Palpate the client's abdomen for bladder distention

CorrectA. Assess the client's three-way urinary catheter for patency

The conflict resolution technique in which each side gives up something and gains something is called: A. Compromising B. Avoiding C. Accommodating D. Competing

CorrectA. Compromising

A nurse is working in an environment where a large portion of the clients appear to be of the same race; however, the nurse is aware that within each broad category of race are numerous cultural groups. Why is this important for the nurse to recognize? A. Different cultural groups have different views of health-related illness practices for care B. To prevent racism from occurring on the unit among the patients C. For demographics data collection D. To provide the best care possible according to thier nertiage and traditions

CorrectA. Different cultural groups have different views of health-related illness practices for care

The unit secretary is sending an e-mail message to all staff. Recognizing e-mail etiquette, the secretary would do which of the following? A. Include a joke at the end of the e-mail so readers will be more likely to read the entire message B. Refrain from forwarding e-mail messages from others without their permission C. Respond immediately to an angry message D. Use capital letters generously for emphasis

CorrectB. Refrain from forwarding e-mail messages from others without their permission

The 18-year-old client diagnosed with renal trauma is admitted to the critical care unit after a serious motor vehicle accident resulting from driving under the influence. The mother comes to the unit and starts yelling at her son about "driving drunk"/ Which action should the nurse implements? A. Escort the mother to a private area and talk with her B. Notify the hospital security to remove the mother C. Allow the mother to continue talking to her son D. Tell the mother if she wants to stay, she must be quiet

CorrectA. Escort the mother to a private area and talk with her

The nurse is engaged in a process approach of reviewing, interpreting, critiquing, and evaluating research and other relevant literature for direct application to patient care. When asked by a coworker, the nurse explains that this process is called: A. Evidence-based care B. Evidence-based practice C. Evidence-based practice D. Critical thinking

CorrectA. Evidence-based care

The nurse is planning the care of a client diagnosed with acute gastroenteritis. Which nursing problem is priority? A. Fluid and electrolyte imbalance B. Altered nutrition C. Impaired body image D. Self-care deficit

CorrectA. Fluid and electrolyte imbalance

Which of the following is the change theory proposed by Lewin? A. Force-field model B. Phases of change C. Diffusion of innovations theory D. Six-step change model

CorrectA. Force-field model

Reward power and coercive power are commonly used to influence others. The effective use of rewards for staff nurses is all of the following except: A. Increased workload for displaying excellence in patient care outcomes B. Increased salary for obtaining the next rung on the clinical ladder C. One additional paid day off for each year you are with the organization D. Formal recognition before one's nursing peers at an awards ceremony

CorrectA. Increased workload for displaying excellence in patient care outcomes

The nurse and the CNA are caring for clients on a medical/surgical unit. Which task should not be assigned to the CNA? A. Instruct the CNA to feed the 69-year-old client with a new diagnosed with dysphagia B. Ask the CNA to obtain vital signs on the 73-year-old client diagnosed with cirrhosis C. Request the CNA change the linens for the 89-year-old client with fecal incontinence D. Tell the CNA to assist the 54-year-old client with a bowel management program

CorrectA. Instruct the CNA to feed the 69-year-old client with a new diagnosed with dysphagia Answers:

The primary nurse overhears the CNA telling a family member of a client, "One of the clients will be going to prison because that person was charged with vehicular manslaughter and two people in the motor vehicle accident died". Which action should the charge nurse take? A. Interrupt the conversation and tell the CNA to report to the nursing station B. Allow the CNA to complete the conversation and then discuss the situation C. Immediately tell the CNA that her comment is a violation of HIPPA D. Apologize to the family member for the CNA's comments

CorrectA. Interrupt the conversation and tell the CNA to report to the nursing station

The nursing instructor has just completed a lecture on evidence-based practice that included a discussion of the different phases. Which response by a nursing student who is identifying the phases would indicate to the nursing instructor that further clarification is needed? A. Making the correct diagnosis B. Locating the best practice C. Clinical application of the best practice findings D. Evaluation of the effectiveness of the care given

CorrectA. Making the correct diagnosis

A department manager's budget concerned with the income and expenses associated with day-to-day activities of hte unit is a(n): A. Operational budget B. Construction budget C. Balanced budget D. Capital budget

CorrectA. Operational budget

A client is seen at the local health center where a flat rate of payment up front is required, instead of reimbursing the health care provider's cost. The client is most likely using which form of payment? A. Prospective payment system B. Cost plus C. Selective payment D. Pay for performance

CorrectA. Prospective payment system

Perceptions of the nurse's role in health care can vary according to culture. For example, In some Asian cultures, when a nurse assists with bathing or feeding, the family may perceive the nurse's actions as which of the following? A. Rude behavior B. Demonstration of attentiveness to the patient's physical comfort needs C. Dedication to the patient D. Respect and an attempt to help the patient

CorrectA. Rude behavior

Nurses use informatics in order to: A. Substitution, innovation, and transformation of patient care, nursing administration, or educational preparation B. Provide management training for nurses interested in communication technology C. Play computer games for entertainment D. Foster collaboration among nurses and others who are interested in nursing informatics

CorrectA. Substitution, innovation, and transformation of patient care, nursing administration, or educational preparation

Which client should the medical unit nurse assess first after receiving the shift report? A. The 84-year-old client diagnosed with pneumonia who is afebrile but getting restless and confused B. The 56-year-old client diagnosed with a left-sided hemothorax with tidaling in the water-seal compartment of the Pleurvac C. The 38-year-old client diagnosed with a sinus infection who has green drainage from the nose D. The 25-year-old client diagnosed with influenza who is febrile and has a headache

CorrectA. The 84-year-old client diagnosed with pneumonia who is afebrile but getting restless and confused

The nurse and CNA are caring for clients on a surgical unit. Which action by the CNA warrents immediate intervention? A. The CNA assists a client who received an IV narcotic analgesic 30 minutes ago to ambulate in the hall B. The CNA empties the indwelling catheter bag for the client with transurethral resection of the prostate (TURP) C. The CNA provides apple juice to the client with a nephrectomy who has just been advanced to a clear liquid diet D. The CNA applies moisture barrier cream to the elderly client with urinary incontinence who has an excoriated perianal area

CorrectA. The CNA assists a client who received an IV narcotic analgesic 30 minutes ago to ambulate in the hall

A nursing instructor determines that the nursing students understand the concept of "knowledge worker" if the students describe which of the following tasks of the knowledge worker? Select all that apply A. Focus on personal, life-long goals and achievement B. Represent the organization C. Provide service D. Bring expert knowledge

CorrectB. Represent the organization CorrectC. Provide service CorrectD. Bring expert knowledge

The CNA is applying elastic compression stockings to the client. Which action by the CNA indicates to the nurse the CNA understands the correct procedure for applying the elastic compression stockings? A. The CNA had the client elevate the legs prior to putting on the stockings B. The CNA applies the stockings while the client is sitting in a chair C. The CNA is unable to insert two fingers under the proximal end of the stockings D. The CNA places the toe opening of the elastic stocking on top of the client's foot

CorrectA. The CNA had the client elevate the legs prior to putting on the stockings

Which client should the telemetry nurse assess first after receiving the AM shift report? A. The client diagnosed with congestive heart failure who has pink frothy sputum B. The client diagnosed with mitral valve stenosis who has heart palpitations C. The client diagnosed with arterial occlusive disease who has intermittent claudication D. The client diagnosed with deep vein thrombosis who has an edematous right calf

CorrectA. The client diagnosed with congestive heart failure who has pink frothy sputum

Which client should the nurse assess first after receiving the p.m. shift report? A. The client with abdominal pain who has a 8-hour urinary output of 150ml/hour B. The client with proctitis who has tenesmus and passage of mucus through the rectum C. The client with liver failure who is jaundiced and has ascites D. The client with Barrett's esophagus who has dysphagia and pyosis

CorrectA. The client with abdominal pain who has a 8-hour urinary output of 150ml/hour

The charge nurse o the critical care respiratory unit is evaluating arterial blood gas (ABG) values of several clients. Which client would require an immediate intervention by the charge nurse? A. The client with reactive airway disease with a pH 7.48, PaO2 80, Pa CO2 30, and HCO3 23. B. The client with a pneumothorax with a pH 7.41, PaO2 98, PaCO2 43, HCO3 25 C. The client with chronic obstructive pulmonary disease who has a pH 7.34, paO2 70, PaCO2 55, HCO3 24 D. The client with Adult Respiratory Distress Syndrome who has a pH 7.35, PaO2 75, PaCO2 50, HCO3 26

CorrectA. The client with reactive airway disease with a pH 7.48, PaO2 80, Pa CO2 30, and HCO3 23.

The nurse on a medical unit has a client with adventitious breath sounds, but the nurse is unable to determine the exact nature of the situation. Which multidisciplinary team member should the nurse consult first? A. The respiratory therapist B. The case manager C. The healthcare provider D. The unit manager

CorrectA. The respiratory therapist

The client scheduled for a D&C is upset because the HCP told her she has syphilis. The client asks the nurse, "This is so embarrassing. Do you have to tell anyone about this?" Which statement is the nurse's best response? A. This must be reported to the Public Health Department and your sexual partners B. You really should tell your sexual partners, so they can be treated for syphilis C. I realize you are embrassed. Would you like to talk about the situation? D. According to the Health Insurance Portability and Accountability Act (HIPAA), I cannot report this to anyone without your permission

CorrectA. This must be reported to the Public Health Department and your sexual partners

The plan-do-study-act model begins with: A. Three questions B. Two concepts C. Five agendas D. Four stages

CorrectA. Three questions

A nurse manager tells the staff that there will be an increasing use of nursing informatics in the clinical setting. The nurse manager explains that, according to the 1998 definition formulated by the International Medical Informatics Association-Nursing Informatics, nursing informatics is defined as: A. A formal educational program in nursing informatics or a graduate program with a nursing informatics focus by an institution of higher learning B. The integration of nursing, its information, and information management with information processing and communication technology to support the health of people worldwide C. A designation applied to any nurses who have successfully educated themselves using formal and/or informal resources and taken a credentialing test for specialty designation D. Differentiated practice representative of the specialty and recognized for certification by at least one organized body through external testing

CorrectB. The integration of nursing, its information, and information management with information processing and communication technology to support the health of people worldwide Answers:

The nurse is adminstering medications to clients on a surgical unit. Which medication should the nurse administer first? A. The proton-pump inhibitor pantoprazole (Protonix) IVPB to the client who is at risk for developing a stress ulcer B. the narcotic analgesic morphine IV infusion to the client who is 8 hours postoperative and is complaining of pain, rating it as a 7 on a 1 to 10 scale C. The loop-diuretic furosemide (Lasix) IVP to the client who has undergone surgical debridement of the right lower limb D. The aminoglycoside antibiotic vancomycin IV piggyback (IVPB) to the client with an infected adbominal wound

CorrectB. the narcotic analgesic morphine IV infusion to the client who is 8 hours postoperative and is complaining of pain, rating it as a 7 on a 1 to 10 scale

A nurse manager who held a commission in the Army was notified of mandatory military duty and had to go to Irag to serve for 2 years. After discharge, the nurse returned to the hospital to resume the previous position but was informed that the position was no longer available. The hospital then offered the nurse employment as a staff nurse, which involved a major reduction in salary. The hospital was in violation of which of the following? A. 1935 National Labor Relations Act (Wagner Act) B. 1973 vietnam Veterans Act C. 1974 Taft-Hartley Amendments to the Wagner Act D. 1964 Civil Rights Act

CorrectB. 1973 vietnam Veterans Act

The SBARR tool is an effective method for communication between nurses and health care practitioners. Which of the following is not correct regarding SBARR? A. S=situation B. A= advice C. B= background D. R=recommendation

CorrectB. A= advice

Three methods to deal with anger, but which may have potentially destructive drawbacks, are listed below. Which is not necessarily correct? A. Denying and repressing anger B. Aggression and confusion C. Turning the other cheek D. Expressing anger

CorrectB. Aggression and confusion

Intellectual capital can be defined as: A. An organization's collective information, which is in written, electronic, or cryptic format B. An individual's knowledge, skills, and abilities that have value and portability C. Work created by an individual but owned solely by the organization D. Ideas and creations formulated at work and sold for profit by the organization

CorrectB. An individual's knowledge, skills, and abilities that have value and portability

A new graduate nurse is assigned a patient who is two days postoperative and has had a colostomy. The patient has an order to have a nasogastric tube inserted immediately. The new graduate has never inserted this type of tube in a patient. How should the new graduate nurse proceed in this situation? A. Delegate the task to a CNA B. Ask an experienced RN for assistance with the procedure C. Read over the procedure, and then insert the tube D. Notify the Dr. of thenew graduate's inexperience

CorrectB. Ask an experienced RN for assistance with the procedure

The CNA reports to the nurse the client's urine output has birght red blood. Which intervention should the nurse implement first? A. Instruct the CNA to take a urine specimen to the laboratory B. Assess the client's urine specimen and complete a renal assessment C. Document the findings in the client's nursing notes D. Ask the CNA to take the client's vital signs

CorrectB. Assess the client's urine specimen and complete a renal assessment

A patient has an advanced directive that indicates no CPR should be performed under any circumstance. When the nurse enters the patient's room and finds that he has coded, the nurse immediately beings CPR. The nurse is at risk for which of the following? A. Negligence B. Battery C. Assault D. Licensure

CorrectB. Battery

A nurse using the writings of Florence Nightingale, would most likely base nursing care on which of the following definitions of "health"? A. A state of complete physical, social, and mental well-being and not merely the absence of disease or infrimity B. Bring well and using every power the individual possesses to the fullest extent C. A state or a process of being and becoming an integrated and whole person D. The state of being free fromillness or injury

CorrectB. Bring well and using every power the individual possesses to the fullest extent

The nurse assesses ventricular fibrilation electrical activity on the telemetry reading while the client is talking to the nurse on the intercom system. Which task should the nurse instruct the CNA to implement? A. Remove the telemetry monitor B. Check the client's telemetry leads C. Find the nurse to check the client D. Call a Code BLue immediately

CorrectB. Check the client's telemetry leads

You and other nurses in your organization have chosen to act as a group with a single voice when dealing with some workplace problems. This is known as: A. Collective bargaining B. Collective action C. Workplace advocacy D. Unionization

CorrectB. Collective action

A number of important skills pertaining to communication have been defined to assist nurses with effective communication. Which of the following is not one of these important identified skills? A. Responding B. Commenting C. Attending D. Clarifying

CorrectB. Commenting

Three requirements for successful teams are; A. Creative environment, like disciplines, and socialization B. Conducive physical, social, and political environment C. Compatible members, environment, and goals D. Compliant administration, political environment, and status

CorrectB. Conducive physical, social, and political environment

It is important for nurses to be asare of generational differences when caring for individuals. Baby boomers are known for their: A. Paying dues and conformity B. Defining themselves through their employment. C. Being comfortable with technology D. Independence and changing employment places often

CorrectB. Defining themselves through their employment.

A nurse is in a situation where there is a conflict between two ethical principles. The nurse must make a decision, but there seems to be no "correct" decision. The nurse is experiencing which of the following? A. Ethics B. Ethical dilemma C. Bioethics D. Ethical situation

CorrectB. Ethical dilemma

The elderly female client diagnosed with osteoporosis is prescribed the bisphosphonate medication alendronate (Fosamax). Which intervention is priority when administering this medication? A. Administer the medication first thing in the morning B. Have the client remain upright for 30 minutes after administering the medication C. Ask the client whether she has a history of peptic ulcer disease D. Encourage the client to walk for at least 30 minutes

CorrectB. Have the client remain upright for 30 minutes after administering the medication

While caring for a population, the nurse recognizes that differences in health risks and health status that reflect the groups' poor health status are known as: A. Minority health B. Health disparities C. Illness prevention D. Health promotion

CorrectB. Health disparities

A nurse suggests that deploying rapid response teams and preventing central line infections are important ideas to implement. These ideas were identified in whihc publication/campaign? A. Joint Commission Patient Safety Goals B. IHI's 100,000 Lives Campaign C. IOM's "To Err Is Human" D. IOM's "Crossing the Quality Chasm"

CorrectB. IHI's 100,000 Lives Campaign

A number of trends have been identified that impact communication, especially health care communication. Which of the following is not one of these identified trends? A. Increasing diversity B. More nurses C. Technology D. Aging population

CorrectB. More nurses

The majority of learning theories fall into three categories. Which is correct? A. Behavioral, psychosocial, and affective B. Perception, information processing, and personality C. Cognition, perception, and information D. Psychomotor, cognitive, and affective

CorrectB. Perception, information processing, and personality

The nurse on a cardiace unit is discussing a client with the case manager. Which information should the nurse share with the case manager? A. Ask the case manager to get the client's permission before sharing information B. Provide the case manager with any information that is required for continuity of care C. Discuss personal information the client shared with the nurse in confidence D. Explain that client confidentiality prevents the nurse from disclosing information Question 11

CorrectB. Provide the case manager with any information that is required for continuity of care

QSEN is an initiative focused on: A. Improving patient satisfaction in the areas of safety and wellness B. Reform in nursing education in the areas of quality and safety C. Statistics for nurses working in environmental health care D. Computer literacy for nurses

CorrectB. Reform in nursing education in the areas of quality and safety

When using available evidence, the nurse must be aware of what two major challenges? A. Balancing the benefits and harm to justify a recommendation of the evidence B. Reviewing the body of knowledge and evaluating for clinical decision making C. Reviewing the clinical trials and grading the research according to the AHRQ scale D. Recommending or not recommending the research evidence for clinicla practice

CorrectB. Reviewing the body of knowledge and evaluating for clinical decision making

The nurse in the labor and delivery department is caring for a client whose abdomen remains hard and rigid between contraction and the fetal heart rate is 100. Which client problem is priority? A. Fluid and electrolyte imbalance B. Risk for fetal demise C. Alteration in comfort D. Inefective breathing pattern

CorrectB. Risk for fetal demise

Angry at a classmate, a nursing student spreads a rumor that the classmate is HIV postitive. The nursing student can be charged with which of the following? A. Battery B. Slander C. Libel D. Assault

CorrectB. Slander

A new graduate wants to explore the three components of each health care system before applying for a position. The graduate would plan to explore which of the following? A. Process, strategy, and opportunity B. Structure, process, and outcome C. Outcome, procedure, and structure D. Strategy, outcome, and performance

CorrectB. Structure, process, and outcome

The nurse is working in the ER of a children's medical center. Which client should the nurse assess first? A. The 2-year-old toddler who was bitten by another child at the day-care center B. The 6-year-old school-age child who was hit by a car while riding a bicycle C. The 14-year-old adolescent whose mother suspects her child is sexually active D. The 1-month-old infant who has developed colic and is crying

CorrectB. The 6-year-old school-age child who was hit by a car while riding a bicycle

The charge nurse on a busy 36-bed rehabilitation unit must send one staff member to the emergency department. Which staff member is the most appropirate person to send? A. The RN who has been employed on the rehab unit for 8 years B. The RN who transferred to the rehabilitation unit from the medical unit C. The CNA who is completing the 4-week orientation to the rehab unit D. The LVN who has worked on the rehab unit for 3 years

CorrectB. The RN who transferred to the rehabilitation unit from the medical unit

The nurse on the cardiac unit is preparing to administer medications after receiving the morning change of shift report. Which medication should the nurse administer first? A. The calcium-channel blockert to the client who has a blood pressure of 110/68 B. The antidysrhythmic to the client in ventricular fibrillation C. The loop diuretic to a client with a serum K level of 3.2 mEq/L D. The cardiac glycoside to the client who has an apical pulse of 58

CorrectB. The antidysrhythmic to the client in ventricular fibrillation

The nurse has recieved the morning shift report on a surgical unit in a community hospital. Which client should the nurse assess first? A. The client who is 2 days postoperative for an emergency appendectomy and is complaining of abdominal pain, rating it as an 5 on a pain scale of 1 to 10. B. The client who is 1 day postoperative for abdominal surgery and has a rigid, hard abdomen C. The client who is 6 hours postoperative small bowel resection who has hypoactive bowel sound in all four quadrants D. The client who is scheduled for an abdominal-peritoneal resection this afternoon and is crying and upset

CorrectB. The client who is 1 day postoperative for abdominal surgery and has a rigid, hard abdomen

The nurse is caring for the following clients on a medical unit. Which client should the nurse assess first? A. The client with renal calculi who is complaiing of flank pain rated as a 4 on scale of 1 to 10 B. The client with BPH (benign prostatic hypertrophy) who has blood oozing from the intravenous site C. The client with nephrotic syndrome who has proteinuria and hypoalbuminema D. The client with acute glomerulonephritis who has oliguria and periorbital edema

CorrectB. The client with BPH (benign prostatic hypertrophy) who has blood oozing from the intravenous site

A patient's son has medical power of attorney and has arrived at the patient's bedside to discuss care options with her. The patient has just been dialyzed, has not received any pain medication, and is rational in her decision making. The parient's sound decides that she should to receive any more dialysis treatments due to the acute state of her illness, the discomfort that she suffered, and her inability to care for herself. The patient disagrees. Which decision should be followed? A. Both, and the hospital ethics committee should convene and decide B. The patient C. The patient's son D. Neither, this is a legal decision and should be done by a court of law

CorrectB. The patient

A disadvantage of functional nursing is: A. The patient receives care from several staff members B. The patient receives task-focused care C. It serves a large number of patients D. It utilizes different skill levels to deliver care

CorrectB. The patient receives task-focused care

Which action by the female primary nurse would warrent immediate intervention by the charge nurse on the rehab unit? A. The primary nurse tells the CNA to escort a client to the swimming pool B. The primary nurse asks another nurse to administer an injection she prepared C. The primary nurse requests another nurse to watch her clients over her lunch break D. The primary nurse evaluates the client's plan of care iwth the family

CorrectB. The primary nurse asks another nurse to administer an injection she prepared

"A set of related common-sense skills that helps people to use their time in the most effective and productive way possible" is the definition of: A. Setting priorities B. Time management C. Making assignments D. Time

CorrectB. Time management

A student asks a nurse manager why they should use a SWOT analysis. The nurse manager explains that a SWOT analysis is: A. A human resource process B. Useful in strategic planning C. A financial management term D. An acronym for a type of quality improvement technique

CorrectB. Useful in strategic planning

A staff nurse asks the nurse manager, "How can I tell if our organization provides culturally inclusive health care?" The most appropriate response by the manager is that the organization: A. Has more men than women in the health care workforce B. Will focus on serving two to three minority populations only C. Includes multiple methods of providing health care D. Uses standard interventions for all populations as a safeguard

CorrectC. Includes multiple methods of providing health care

A nursing instructor is evaluating a student's understanding of the primary differences between QA and QI. Which response would indicate that the student understood? A. QI is more a single program, and QA is more a management approach B. QI is reactive, and QA is proactive C. QA's emphasis is on maintaining minimum standards of care, and QI's emphaisisis upon identifying real and potential problems D. QA documents quality, and QI reports incidents and errors

CorrectC. QA's emphasis is on maintaining minimum standards of care, and QI's emphaisisis upon identifying real and potential problems

How long are candidates given to complete the NCLEX-RN exam? A. 4 hours B. 3 hours C. 6 hours D. 5 hours

CorrectC. 6 hours

A nurse manager states to the staff nurses, "This group has such synergy!" One of the new nurses asks, "What does that mean/" The most appropriate response by the nurse manager is that synergy is: A. A philosophical term relating to sharing of visions and goals B. A stage of team development C. A bonus when things work together harmoniously D. When luck and circumstance work together

CorrectC. A bonus when things work together harmoniously

The nurse educator explains to the nursing student that an evidence-based practice (EBP) guideline is: A. Used in medical credentialing B. A guidelilne fo rassessing practice C. A standardized specification for care of a typical patient in the typical situation. D. A descriptive tool for identifying specified care in unique situations

CorrectC. A standardized specification for care of a typical patient in the typical situation.

The 8-year -old client diagnosed with a vaso-occusive sicle cell crisis is complaining of a very severe headache. Which intervention should the nurse implement first? A. Administer a narcotic analgesic by intravenous push (IVP) B. Increase the client's intravenous (IV) rate C. Assess the client's neurological status D. Administer 6 L of oxygen via nasal cannu

CorrectC. Assess the client's neurological status

The nurse is caring for clients on a medical unit. Which task should the nurse implement first? A. Change the abdominal surgical dressing for a client who has ambulated in the hall B. Place a call to the extended care facility to give the report on a dsicharged client C. Assess the male client who called the desk to say he is nauseated and just vomited. D. Discuss the correct method of placing Montgomery straps on the client with the CNA

CorrectC. Assess the male client who called the desk to say he is nauseated and just vomited.

The charge nurse on the renal unit is notified of a bus accident with multiple injuries and clients are being broght to the ER. The hospital is implementing the disaster policy. Which action should the nurse take first? A. Call all off-duty nurses to notify them to come in to work B. Request all visitors to leave the hospital as soon as possible C. Assess the staffing to determine which staff could be sent to ER if needed D. Determine which clients could be discharged home immediately

CorrectC. Assess the staffing to determine which staff could be sent to ER if needed

The Quality Improvement Team has begun assessing and analyzing the care given to TB patients. This is an example of which organizational strategy for quality and process improvement? A. Using a storyboard B. Meeting regulatory requirements C. Benchmarking D. Identifying opportunties for system change following a sentinel event review

CorrectC. Benchmarking

A staff nurse is speaking with a nurse manager regarding politics and nursing. The nurse manager explains that politics exists in any system where which of the following conditions exists? A. Unlimited distribution of resources B. Abundant resources C. Competing interests for resources D. Unlimited control of resources

CorrectC. Competing interests for resources

An instructor has just completed teaching a class on critical thinking to a group of sophomore nursing students. On the test the instructor asked, "Which of the following are the basic components required for the development of sound critical thinking skills?" Which of the following responses would indicate that further teaching is needed? A. Critical listening B. Critical writing C. Critical Evaluation D. Critical reading

CorrectC. Critical Evaluation

Your supervisor comments that you have a great deal of personal power. The supervior's comment is most likely related to which of your assets? A. Charisma when speaking to people B. Expertise as a nurse C. Decision-making ability D. Connections to other nurses

CorrectC. Decision-making ability

Which of these general principles of total quality management does not necessarily belong? A. Quality is achieved by the participation of everyone B. Focusing on the work process develops improvement opportunities C. Decisions to improve or change a process are based on the majority role D. Improving the service of quality is a continuous process

CorrectC. Decisions to improve or change a process are based on the majority role

The nurse is inserting an indwelling catheter into a male elderly client. Which intervention should the nurse implement first? A. Ensure urine is obtained in the indwelling catheter B. Ask the client if he has any prostate problems C. Determine if the client has any betadine allergies D. Lubricate the end of the indwelling catheter

CorrectC. Determine if the client has any betadine allergies

The client is in the cardiac intensive care unit on dopamine, a vasoconstrictor, and the B/P increaess to 210/130. Which intervention should the intensive care nurse implement first? A. Notify the client's healthcare provider B. Administer the vasopressor hydralazine C. Discontinue the client's vasoconstrictor,dopamine D. Assess the client's neurological status

CorrectC. Discontinue the client's vasoconstrictor,dopamine

Some methods for creating more time are listed below. Which is not necessarily correct? A. Delegate work B. Get up one hour earlier C. Eliminate tasks you do not like D. Prioritize work

CorrectC. Eliminate tasks you do not like

The degree to which people perceive they have power over their environment is: A. Control B. Fate and Luck C. Environmental control D. Transcultural delegation

CorrectC. Environmental control

A health care organization espouses beneficence, nonmaleficence, fidelity, justice, autonomy, respect for others, and veracity. A nurse employed by the organization understands that these are all examples of which of the following? A. Philosophies related to organizational law B. Philosophies pertaining to nursing C. Ethical principles and rules D. Ethical dilemmas

CorrectC. Ethical principles and rules

In the role of patient advocate, you would: A. Act as a proxy for the patient in understanding and signing important consents B. Ensure the patient does not make risky health decisions C. Interpret the health care environment for culturally diverse patients D. Educate the patient to ensure compliance with the health care organization's goals

CorrectC. Interpret the health care environment for culturally diverse patients

A nursing instructor has just completed teaching the nursing students about the six areas of health needed for a holistic approach to good healthy living. On the quiz the instructor asked the students to identify the six areas. Which answer by the students would indicate to the instructor that additional teaching is needed? A. Emotional B. Physical C. Interrelationships D. Professional

CorrectC. Interrelationships

There is a going-away party for one of the nurses, who has been promoted to a managerial position in another hospital that is part of the same health care organization. Photographs have been taken of nurses around the unit as a way for the nurse to remember fellow colleagues. If patients are inadvertently photographed and no consent for this is obtained, this is an example of: A. Defamation B. Assault C. Invasion of privacy D. Battery

CorrectC. Invasion of privacy

A nurse manager is composing a vision statement for the organization. The nurse should NOT include which of the following of the four elements of a sound vision statement? A. It is written in the present tense using action words B. It balances the needs of providers, patients, and the environment C. It covers a variety of activities and spans specific short-term time frames D. It is written down

CorrectC. It covers a variety of activities and spans specific short-term time frames

The client who is 1 day postoperative following chest surgery is having difficulty breathing, has bilateral rales, and is confused and restless. Which intervention should the nurse implement first? A. Check the client's surgical dressing B. Assess the client's pulse oximeter reading C. Notify the Rapid Response Team D. Place the client in Trendelenburg position

CorrectC. Notify the Rapid Response Team

A nurse wants to gain a better understanding of the organization's structure and lines of communication. The nurse should begin by reviewing which of the following? A. Decentralization pattern B. Hierarchical structure C. Organizational chart D. Centralization pattern

CorrectC. Organizational chart

There are numberous factors that contribute to the rising number of uninsured in the United States. Which is not necessarily a contributing factor? A. Higher premiums B. People between jobs or umemployed C. People being eligible for public programs D. Employers not offering health insurance

CorrectC. People being eligible for public programs

The nurse manager is developing the value statement of the principles and beliefs that direct the behavior on the unit. The value statement is consistent with that of the organization and will become the unit's: A. Mission B. Strategic Plan C. Philosophy D. Purpose

CorrectC. Philosophy

A nurse manager is engaged in a process designed to achieve goals in dynamic, competitive environments through allocation of resources. This process is called: A. Total Quality Improvement B. Process improvement C. Strategic planning D. Market analysis

CorrectC. Strategic planning

The nurse is caring for clients on the pediatric medical unit. Which client should the nurse assess first? A. The child diagnosed with fastroenteritis who has a potassium (K+) level of 3.9 mEq/L B. The child diagnosed with type 1 diabetes who has a blood glucose level of 180mg/dL C. The child diagnosed with cystic fibrosis who has a pulse oximeter reading of 90% D. The child diagnosed with pneumonia who is coughing and has a temperature of 100F

CorrectC. The child diagnosed with cystic fibrosis who has a pulse oximeter reading of 90%

A nurse manager is discussing with a colleague the fact that in 2003 the American Hospital Association replaced the Patient Bill of rights with a document entitled the Patient Care Partnership. Which of the following statements regarding what has been included in the new document is correct? A. The patient has the right to review the records of themselves and thier family members B. The patient has the right to have an advance directive concerning power of attorney for business decisions C. The patient has the right to high quality hospital care;safe environment; and involvement in thier own care. D. The patient has the right to considerate and immediate care

CorrectC. The patient has the right to high quality hospital care;safe environment; and involvement in thier own care.

The plan-do-study-act (PDSA) cycle begins with: A. Five agendas B. Four stages C. Three questions D. Two concepts

CorrectC. Three questions

The client in the post-anesthesia care unit (PACU) has noisy and irregular respirations with a pulse oximeter reading of 89%. Which intervention should the PACU nurse implement first? A. Increase the client's oxygen rate via nasal cannula B. Obtain an intubation tray and prepare for emergency intubation C. Tilt the head back and push forward on the angle of the lower jaw D. Notify the respiratory therapist to draw arterial blood gases

CorrectC. Tilt the head back and push forward on the angle of the lower jaw

A new graduate nurse asks the charge nurse, "How do I know what I can and cannot delegate?" What is the best reply? A. "If you follow the Five Rights of Delegation, you will never have an issue. B. "You will not be delegating, only the charge nurse delegates, so there is nothing to worry about." C. "Delegation takes time and practice to learn what you can and cannot do." D. "Every state has standards of practice, laws on scope of practice, and guidelines for delegation to guide you."

CorrectD. "Every state has standards of practice, laws on scope of practice, and guidelines for delegation to guide you."

The staff developer has just completed a session on cultural diveristy and nursing care. One of the questions on the final exam asks the participants about their feeling related to caring for cients from different ethnic grops than the nurse's. The staff developer is measuring which domain of learning? A. Psychomotor B. Cognitive C. Effective D. Affective

CorrectD. Affective

A new nursing graduate is working on a unit project regarding patient care and GI treatments. While interviewing a patient, who has been incessantly complaining about his lack of daily bowel movements (complete with graphic details about the type and consistency), the new nursing graduate tells the patient that if he is not quiet she will give him an enema he will never forget. This is an example of: A. Invasion of privacy B. Deflamation C. Battery D. Assault

CorrectD. Assault

A nurse intern asks the unit manager how nurses can become more politically involved in the consumer movement in health care to support the advancement of nursing service. The nurse manager responds by saying that the majority of nurses can accomplish this by: A. Becoming a lobbyist to interested government agencies B. Supporting legislation for more government control C. Supporting physicians to direct health care policy D. Becoming a member of their professional organization

CorrectD. Becoming a member of their professional organization

Some predictors of performance on the NCLEX-RN exam are listed below. Which is not necessarily correct? A. HESI exit exam B. ACT score C. Absence of emotional distress D. Confidence in one's abilities

CorrectD. Confidence in one's abilities

An advantage of the factor-type patient classification system is: A. Ongoing workload for nurses to classify patients B. Capturing typical nursing time C. Capturing holistic patient needs D. Data are readily available

CorrectD. Data are readily available

A nurse manager who uses a leadership style that is participatory and where authority is delegated to others is most likely using which of the following leadership styles? A. Laissez-faire B. Autocratic C. Employee-centered D. Democratic

CorrectD. Democratic

According to Vroom's Theory of Motivation, force: A. Describes people who have free will but choose to comply with orders they are given B. Is the perceived possibility that the goal will be achieved C. Is a naturally forming social group that can become a contributor to an organization D. Describes the amount of effort one will exert to reach one's goal

CorrectD. Describes the amount of effort one will exert to reach one's goal

The nurse manager understands that one of the purposes of strategic planning is to: A. Concentrate on higher productivity B. Eliminate competitive threats C. Emphasize a marketing strategy D. Develop a coordinated vision

CorrectD. Develop a coordinated vision

A nurse working in a private physician's office noted that several of the clinic patients had been given prescriptions for cholesterol medications even though the laboratory results did not indicate the need for these medications. Concerned with the patient's health and safety, the nurse reported her suspicions to the proper authority. The nurse's actions are based on which piece of legislation which encourages individuals to come forward to report a danger to public health or safety? A. Anti-fraud Act B. Taft-Hartley Amendments to the Wagner Act C. Edmunds-Tucker Act D. False Claims Act Modification

CorrectD. False Claims Act Modification

When planning the budget for the unit, the nurse manager recognizes that equipment depreciation, utilities, fringe benefits, and salaries are considered: A. Variable costs B. Indirect expenses C. Direct expenses D. Fixed costs

CorrectD. Fixed costs

The nurse recognizes that there are certain variables that increase or decrease the probability of illness or death, and that these variables can be modified. Such variables are called: A. Health determinants B. Vulnerable population groups C. Underserved D. Health risk factors

CorrectD. Health risk factors

The nurse manager recognizes that the goal of studying outcomes is to: A. Incorporate change in nursing practice B. Determine staff needs C. Predict the quality of patient care D. Identify potential problems

CorrectD. Identify potential problems

A nursing instructor has asked a group of students to develop teaching plans for their clients according to Gagne's nine elements. If the instructor asks the students about some of the components of Gagne's events of instruction, which response by a student would indicate that further teaching is needed? A. Provide feedback B. Gain attention C. Present stimulus materials D. Implement performance

CorrectD. Implement performance

A staff nurse asks the nurse manager, "How can I tell if our organization provides culturally inclusive health care?". The most appropriate response by the manager is that the organization: A. Will focus on serving two to three minority populations only B. Has more men than women in the health care workforce C. Uses standard interventions for all populations as a safeguard D. Includes multiple methods of providing health care

CorrectD. Includes multiple methods of providing health care

A new graduate complains to the charge nurse, "I don't see why we have to work in teams." The best response by the charge nurse regarding advantages of teamwork would include: A. Increased interpersonal skills, communication, and delegation of responsibility B. Innovation, collaboration, and socialization C. Collaboration, communication, and job satisfaction D. Increased interprofessional communication, collaboration, and job satisfaction

CorrectD. Increased interprofessional communication, collaboration, and job satisfaction

A nurse delegated the turning of a particular patient to the CNA. After an hour had passed , the nurse asked the CNA if the patient had been turned. The CNA replied,"No". What should the nurse have done initially to ensure that the task was completed in a timely manner? A. Explain the procedure to the CNA B. Assisted the CNA in turning the patient C. Delegated this task to another staff member D. Informed the CNA of the time frame in which the task was to be completed

CorrectD. Informed the CNA of the time frame in which the task was to be completed

A nurse has heard that a fellow nurse at another hospital was charged with assault on a patient. The nurse asks the nurse manager whether assault is the same as battery. The nurse manager explains that assault differs from battery in that it: A. Is the act instead of the threat being done without permission B. Concerns being offensive without permission C. Concerns the thought without the act or the threat D. Is the threat instead of the act being done without permission

CorrectD. Is the threat instead of the act being done without permission

You are organizing your review in order to be prepared for the NCLEX-RN. Which of the following is not necessarily an area to concentrate on for this review? A. Test Anxiety control B. NCLEX knowledge review C. NCLEX test question practice D. NCLEX review/reread of all of your nursing textbooks and talking with teachers

CorrectD. NCLEX review/reread of all of your nursing textbooks and talking with teachers

A group of newly hired nurses ask a nurse manager if they should carry malpractice insurance. The nurse manager tells them that there are a number of good reasons for nurses to carry their own malpractice insurance. Which of the following would NOT be included? A. Their institution's insurance may not cover them if they fail to comply with its policies and procedures B. Nurses are being named individually in lawsuits C. Their institution may fail to cover them if they acted outside the scope of thier practice D. Nurses are being considered easy targets for lawsuits

CorrectD. Nurses are being considered easy targets for lawsuits

During which step in the change process would the who, how, and when of the change be determined? A. Evaluation B. Implementation C. Assessment D. Planning

CorrectD. Planning

The CNA reports to the nurse that a patient is complaining of shortness of breath (SOB). Which task can the nurse delegate to the CNA? A. Listen to the patient's lungs B. Notify the RT that the patient needs a breathing treatment C. Document the condition of the patient since the CNA is the person reporting it D. Position the patient in a high Fowler's position

CorrectD. Position the patient in a high Fowler's position

The Health Insurance Portability and Accountability Act (HIPPA) was developed to: A. Make it easier to transport and transmit your personal health information B. Make health insurance companies more accountable for payment of claims C. Guard against insurance companies selling their client list of names and addresses D. Protect all individuals' identifiable health information held or transmitted

CorrectD. Protect all individuals' identifiable health information held or transmitted

Your organization is planning to make educational and cultural changes. Which model of change is most often used in this type of situation? A. Force-field model B. Phases of change model C. Diffusion of innovations theory of change model D. Six-step change model

CorrectD. Six-step change model

A group of professionals has joined together in a team to address the issue of low morale among employees. During which stage of team formation would interpersonal issues come in to play? A. Norming B. Performing C. Adjourning D. Storming

CorrectD. Storming

Which client should the charge nurse on the rehabilitation unit assess first after receiving the AM shift report? A. The client diagnosed with rheumatoid arthritis who has a positive rheumatoid factor (RF) B. The client diagnosed with an open reduction and internal fixation (ORIF) of the right hip who has a hemoglobin and hematocrit (H&H) of 8/24 C. The client diagnosed with systemic allergies on prednisone dose pack who has a glucose level of 189 mg/dL D. The client diagnosed with a Stage IV pressure ulcer who has a white blood cell (WBC) count of 14,000

CorrectD. The client diagnosed with a Stage IV pressure ulcer who has a white blood cell (WBC) count of 14,000

Which client should the postpartum nurse assess first after receiving the AM shift report? A. The client who is crying because the baby will not nurse B. The client who is refusing to have the newborn in the room C. The client who is complaining of perineal pain when urinating D. The client who has saturated multiple peri-pads during the night

CorrectD. The client who has saturated multiple peri-pads during the night

The nurse has just finished the change-of-shift report. Which patient should the nurse assess first? A. An elderly client who has requested medication for pain B. A client who is being discharged today C. The patient who needs assistance transferring from the bed to the wheelchair D. The client with COPD who is having difficulty breathing

CorrectD. The client with COPD who is having difficulty breathing

The NCLEX-RN exam questions may take a variety of different formats. Which is not a correct format for NCLEX-RN exam questions? A. Fill in the blank B. Multiple choice: single answer C. Identification of a specific area on a picture or graph D. True or false

CorrectD. True or false

When a nurse recommends to the nurse manager a more efficient approach to organizing care, this type of organizational communication is called: A. Downward B. Diagonal C. Lateral D. Upward

CorrectD. Upward

During your transition from student nurse to your first position as a staff nurse, your general unit orientation is a key component of the transition. Which of the following questions would be most appropriate for you to ask about orientation? Answers: A. How will I be paid for my time in orientation? B. Do I have to attend every orientation session? C. Will I be graded during the orientation? D. How long should I expect to be in orientation?

D. How long should I expect to be in orientation?

A nurse writes a cover letter to a potential employer. The nurse recognizes that one of the major components of a cover letter is that it should be: Answers: A. A detailed commercial about yourself B. One to two pages in length C. Started with a general salutation D. One page in length

D. One page in length

Including personal attributes that illustrate your abilities as an effective team player, continuous learner, and consistent performer in your resume show potential employers that you may be a good fit for their organization. Which is NOT necessarily a correct attribute? Answers: A. Attention to detail B. Reliability in attendance and punctuality C. Seeking out learning opportunities D. Resistance in conflict

D. Resistance in conflict Answers:

A number of studies have provided insight into what characteristics comprise a profession. Of the characteristics listed below, which is not necessarily correct? Answers: A. Work based upon systematic body of theory and abstract knowledge B. Existence of a code of ethics C. Autonomy of decision making D. Results can only be standardized over time

D. Results can only be standardized over time

3. A problem-solving approach that integrates best current evidence with clinical practice

Evidence-based practice is defined as: 1. Nursing care based on tradition 2. Scholarly inquiry of nursing and biomedical research literature 3. A problem-solving approach that integrates best current evidence with clinical practice 4. Quality nursing provided in an efficient and economically sound manager

4. The nurse explains the procedure for giving a tube feeding to a second nurse who has floated to the unit to assist with care.

In which of the following examples is the nurse not applying critical thinking skills in practice? 1. The nurse considers personnel experience in performing intravenous (IV) line insertion and ways to improve performance. 2. The nurse uses a fall risk inventory scale to determine a patient's fall risk. 3. The nurse observes a change in a patient's behavior and considers which problem is likely developing. 4. The nurse explains the procedure for giving a tube feeding to a second nurse who has floated to the unit to assist with care.

2. Coach her to give herself positive messages about her ability to do this

Mrs. Jones states that she gets anxious when she thinks about giving herself insulin. How do you use your understanding of intrapersonal communication to help with this? 1. Provide her the opportunity to practice drawing up insulin 2. Coach her to give herself positive messages about her ability to do this 3. Bring her written material that clearly describes the steps of insulin administration 4. Use therapeutic communication to help her express her feeling about giving herself an injection

4. Provides a minimal standard of knowledge for a registered nurse in practice

The examination for registered nurse licensure is exactly the same in every state in the United States. The examination: 1. Guarantees safe nursing care for all patients 2. Ensures standard nursing care for all patients 3. Ensures that honest and ethical care is provided 4. Provides a minimal standard of knowledge for a registered nurse in practice

2. Description of the efforts to restore the child's blood pressure, what was used, ad questions about the child's response 3. The meaning the experience had for the nurse with respect to her understanding of dealing with a patient's death 4. A description of what the nurse said to the mother, the mother's response, and how the nurse might approach the situation differently in the future

The nurse cared for a 14-year old with renal failure who dies near the end of the work shift. The health care team tried for 45 minutes to resuscitate the child with no success. The family was devastated by the loss, and, when the nurse tried to talk with them, the mother said, "You can't make me feel better; you don't know what it's like to lose a child." Which of the following examples of journal entries might best help the nurse reflect and think about this critical experience? (Select all that apply.) 1. Data entry of time of day, who was present, and condition of the child 2. Description of the efforts to restore the child's blood pressure, what was used, ad questions about the child's response 3. The meaning the experience had for the nurse with respect to her understanding of dealing with a patient's death 4. A description of what the nurse said to the mother, the mother's response, and how the nurse might approach the situation differently in the future

2. Leave a night light on in the bathroom. 6. Provide scheduled toileting during the night shift. 7. Keep the pathway from the bed to the bedroom clear.

The nurse found a 68-year old female patient wandering in the hall. The patient says she is looking for the bathroom. Which interventions are appropriate to ensure the safety of the patient? (Select all that apply.) 1. Insert a urinary catheter. 2. Leave a night light on in the bathroom. 3. Ask the physician to order a restraint. 4. Keep the bed in low position with upper and lower side rails up. 5. Assign a staff member to stay with the patient. 6. Provide scheduled toileting during the night shift. 7. Keep the pathway from the bed to the bedroom clear.

3. The patient is short of breath.

The nurse has a patient who is short of breath and calls the health care provider using SBAR (situation-background-assessment-recommendation) to help with the communication. What does the nurse first address? 1. The respiratory rate is 28. 2. The patient has a history of lung cancer. 3. The patient is short of breath. 4. He or she requests an order for a breathing treatment.

4. "I'm going to attend a support group to learn more about multiple sclerosis."

The nurse is evaluating the coping success if a patient experiencing stress from being newly diagnosed with multiple sclerosis and psychomotor impairment. The nurse realizes that the patient is coping successfully when the patient says: 1. "I'm going to learn to drive a car so I can be more independent." 2. "My sister says she feels better when she foes shopping, so I'll go shopping." 3. "I've always felt better when I go for a long walk. I'll do that when I get home." 4. "I'm going to attend a support group to learn more about multiple sclerosis."

1. Role play

The nurse is teaching a parenting class to a group of pregnant adolescents. The nurse pretends to be the baby's father, and the adolescent mother is asked to show how she would respond to the father if he gave her a can of beer. Which teaching approach did the nurse use? 1. Role play 2. Discovery 3. An analogy 4. A demonstration

4. Search for identity with peer groups and separating from family

The nurse plans care for a 16-year old male, taking into consideration that stressors experienced most commonly by adolescents include which of the following? 1. Loss of autonomy caused by health problems 2. Physical appearance, family, friends, and school 3. Self-esteem issues, changing family structure 4. Search for identity with peer groups and separating from family

1. Diagnostic reasoning.

The nurse sits down to talk with a patient who lost her sister 2 weeks ago. The patient reports she is unable to sleep, feels very fatigued during the day, and is having trouble at work. The nurse asks her to clarify the type of trouble. The patient explains she can't concentrate or even solve simple problems. The nurse records the results of the assessment, describing the patient as having ineffective coping. This is an example of: 1. Diagnostic reasoning. 2. Competency. 3. Inference. 4. Problem solving.

2. Clarifying

The nurse states, "When you tell me that you're having a hard time living up to expectations, are you talking about your family's expectations?" The nurse is using which therapeutic communication technique? 1. Providing information 2. Clarifying 3. Focusing 4. Paraphrasing

3. Remove all patients in immediate danger

The nurse's first action after discovering an electrical fire in a patient's room is to: 1. Activate the fire alarm 2. Confine the fire by closing all doors and windows 3. Remove all patients in immediate danger 4. Extinguish the fire by using the nearest fire extinguisher

4. nurse researcher

The nurses on an acute care medical floor notice an increase in pressure ulcer formation in their patients. A nurse consultant decides to compare two types of treatment. The first the procedure currently used to assess for pressure ulcer risk. The second uses a new assessment instrument to identify at-risk patients. Given this information, the nurse consultant exemplifies which career? 1. clinical nurse specialist 2. nurse administer 3. nurse educator 4. nurse researcher

1. Place a bed alarm device on the bed.

The nursing assessment of an 80-year old patient who demonstrates some confusion but no anxiety reveals that the patient is a fall risk because she continues to get out of bed without help despite frequent reminders. The initial nursing intervention to prevent falls for this patient is to: 1. Place a bed alarm device on the bed. 2. Place the patient in a belt restraint. 3. Provide one-on-one observation of the patient. 4. Apply wrist restraints.

4. Risk for falls

The nursing assessment on a 78-year old woman reveals shuffling gait, decreased balance, and instability. On the basis of the patients data, which of the following nurses diagnoses indicates an understanding of the assessment findings? 1. Activity intolerance 2. Impaired bed mobility 3. Acute pain 4. Risk for falls

3. Develop topics for discussion that require problem solving

The school nurse is about to teach a freshman-level high school health class about nutrition. What is the best instructional approach to ensure that the students meet the learning outcomes? 1. Provide information using a lecture 2. Use simple words to promote understanding 3. Develop topics for discussion that require problem solving 4. Complete an extensive literature search focusing on eating disorders

3. Consistent

The surgical unit has initiated the use of a pain-rating scale to assess patients' pain severity during their postoperative recovery. The registered nurse (RN) looks at the pain flow sheet to see the pain scores recorded for a patient over the last 24 hours. Use of the pain scale is an example of which intellectual standard? 1. Deep 2. Relevant 3. Consistent 4. Significant

A resume for a new graduate RN should "sell" their skills not their past emplyment or roles. True or False

True Response Feedback: Working at McDonalds is not what you sell but the skill set you used there.

3. A thorough physical assessment

When assessing an older adult who is showing symptoms of anxiety, insomnia, anorexia, and mild confusion, one of the first assessments include which of the following? 1. The amount of family support 2. A 3-day diet recall 3. A thorough physical assessment 4. Threats to safety in her home

3. 18 inches to 4 feet from the patient.

When the nurse takes the patient's history, he or she sits: 1. Next to the patient. 2. 4 to 12 feet from the patient. 3. 18 inches to 4 feet from the patient. 4. 12 inches to 3 feet from the patient.

2. Nurse practitioner 3. Certified clinical nurse specialist

Which of the following nursing roles may have prescriptive authority in their practice? (Select all that apply.) 1. Critical care nurse 2. Nurse practitioner 3. Certified clinical nurse specialist 4. Charge nurse

2. "Why do you always put so much salt on your food?"

Which of the following statements would be most likely to block communication? 1. "You look kind of tired today." 2. "Why do you always put so much salt on your food?" 3. "It sounds like this has been a hard time for you." 4. "If you use your oxygen when you walk, you may be able to walk farther."

4. Problem solving.

While assessing a patient, the nurse observes that the patient's intravenous (IV) line is not infusing at the ordered rate. The nurse assesses the patient for pain at the IV site, checks the flow regulator on the tubing, looks to see if the patient is lying on the tubing, checks the point of connection between the tubing and the IV catheter, and then checks the condition of the site where the intravenous catheter enters the patient's skin. After the nurse readjusts the flow rate, the infusion begins at the correct rate. This is an example of: 1. Inference. 2. Diagnostic reasoning. 3. Competency. 4. Problem solving.

3. Takes antihypertensive and diuretics 4. History of recent fall 5. Neglect, spatial and perceptual abilities, impulsive 6. Requires assistance with activity, unsteady gait 7. IV line, urinary catheter

You are admitting Mr. Jones, a 64-year old patient who had a right hemisphere stroke and a recent fall. The wife stated that he has a history of high blood pressure, which is controlled by an antihypertensive and a diuretic. Currently he exhibits left sided neglect and problems with spatial and perceptual abilities and is impulsive. He has moderate left-sided weakness that requires the assistance of two and the use of a gait belt to transfer to a chair. He currently has an intravenous (IV) line and a urinary catheter in place. What factors increase his fall risk at this time? (Select all that apply.) 1. Smokes a pack a day 2. Used a cane to walk at home 3. Takes antihypertensive and diuretics 4. History of recent fall 5. Neglect, spatial and perceptual abilities, impulsive 6. Requires assistance with activity, unsteady gait 7. IV line, urinary catheter

4. Talk with him about his favorite hobbies

You are caring for Mr. Smith, who is facing amputation of his leg. During the orientation phase of the relationship, what would you do? 1. Summarize what you have talked about in the previous sessions 2. Review his medical record and talk to other nurses about how he is reacting 3. Explore his feelings about losing his leg 4. Talk with him about his favorite hobbies

3. Move to her bedside, get her attention, and repeat the question while facing her

You are caring for an 80-year old woman, and you ask her a question while you are across the room washing your hands. She does not answer. What is your next action? 1. Leave the room quietly since she evidently does not want to be bothered right now 2. Repeat the question in a loud voice, speaking very slowly 3. Move to her bedside, get her attention, and repeat the question while facing her 4. Bring her a communication board so she can express her needs

4. Defines the principles of right and wrong to provide patient care.

You are participating in a clinical care coordination conference for a patient with terminal cancer. You talk with your colleagues about using the nursing code of ethics for professional registered nurses to guide care decisions. A nonnursing colleague asks about this code. Which of the following statements best describes this code? 1. Improves self-health care 2. Protects the patient's confidentiality 3. Ensures identical care to all patients 4. Defines the principles of right and wrong to provide patient care.

2. "When you brush me off like that, it takes me even longer to do my job."

You ask another nurse how to collect a laboratory specimen. The nurse raises her eyebrows and asks, "Why don't you figure it out?" what would be the best response? 1. Say nothing and walk away. Find a different nurse to help you. 2. "When you brush me off like that, it takes me even longer to do my job." 3. "Why do you always put me down like that?" 4. "I guess I just enjoy having you make fun of me."

All of the following are true about accountability except ___. a. Accountability can be delegated. b. Accountability is invested in individuals, not group. c. Accountability is about the products of work, not the processes of work. d. Accountability implies change.

a. Accountability can be delegated.

Critical influences that are creating shifts in health care include all of the following except ___. a. Change forces are usually just seen on a local level. b. The financial model now reflects a strong emphasis on value rather than the volume. c. Focus on healthcare system structures, processes, and evidence is reflected in dynamics that clearly demonstrate impact, effectiveness, and significant measures of the levels of health. d. Evidentiary dynamics and improvement science now guide mechanisms of advancing practice within the context of a strongly interfaced digital information infrastructure.

a. Change forces are usually just seen on a local level.

Setting the table means ___. a. knowing how all the decisions need to be served b. making all of the decisions for the team c. selecting team members based solely on how well they work together d. limiting the diversity of the group

a. knowing how all the decisions need to be served

All of the following are true about negotiation except ___. a. Effective negotiation is a skill one is born with. b. Negotiation is learned through the process of discipline and application. c. Much of the skill and talent necessary to negotiate well can be improved through continuous and effective use and refinements of skill development and learning. d. In all negotiation it is important to be able to develop and utilize good listening skills.

a. Effective negotiation is a skill one is born with.

Of the following statements ____ is true. a. Optimization in the system is a reflection of the strong goodness of fit between the human organizational dynamics and the technological functional processes that comprise the overall structure of the work environment. b. Complex thinking that focuses on either social or technical processes increases the predictable. c. Health care contains little interaction between social and technical forces.

a. Optimization in the system is a reflection of the strong goodness of fit between the human organizational dynamics and the technological functional processes that comprise the overall structure of the work environment.

The novice to expert skill acquisition model for nursing was developed by ___. a. Pat Benner b. Florence Nightingale c. Madeline Leininger d. Dorothea Orem

a. Pat Benner

The following is true__. a. Physicians cannot delegate physician work to nurses. b. Physicians can delegate physician work to nurses. c. If the activity or task is not within the nurse's scope of practice, it can be delegated by the nurse. d. Delegation is necessary even if the particular activity or task is already within the legally recognized scope of practice of the individual who is to perform the activity or task.

a. Physicians cannot delegate physician work to nurses.

Examples of poor communication in fully sharing personal views, insights, or opinion include all of the following except ___. a. Staff vocalizes affirmatively. b. You see people nodding their heads or sitting in silence in response to a statement or question raised. c. You notice people stop questioning or raising issues because they fear being labeled as not being team players.

a. Staff vocalizes affirmatively.

Autonomy is ___. a. The right to self-determination b. the duty to do good c. a problem that confronts one, with a choice of solutions that seem or are equally unfavorable d. duty to keep one's promise

a. The right to self-determination

The delegator is responsible for all of the following except ____. a. accepting only those tasks or assignments for which he or she is qualified. b. assessment of the situation. c. ascertaining the competence of the delegate. d. follow-up supervision. e. management of results.

a. accepting only those tasks or assignments for which he or she is qualified.

For professionals ____ is the definitive foundation for work. a. accountability b. responsibility c. competence d. functionalism

a. accountability

Variance management is____. a. analysis of the difference between actual hours of staff and required hours of care and then using this information to create effective workload management systems. b. using multiple indicators to evaluate staffing effectiveness. c. assisting less-experienced staff in organizing and providing care. d. eliminating non-value-added work.

a. analysis of the difference between actual hours of staff and required hours of care and then using this information to create effective workload management systems.

When a health care professional performs favors for a patient that are outside the therapeutic relationship, this is called _____. a. boundary violation b. boundary crossing c. ethical fading d. whistle-blowing

a. boundary violation

The historical usage approach to staffing involves___. a. looking at usage from the year before. b. the assumption that all patients are similar in needs. c. identifying nurse-to-patient ratios based on experiences and perceptions of nurses. d. use of a patient classification system.

a. looking at usage from the year before.

Care at the end of life for which there is little hope of benefit is an example of ____. a. medical futility b. organizational integrity c. nonmaleficence d. rationalization

a. medical futility

Underdelegation is when __. a. not enough work is delegated. b. the workload is beyond what the delegate can reasonably do **********igned time. c. the delegator is constantly looking over the shoulders of those asked to do the work. d. any of the five rights are violated.

a. not enough work is delegated.

Social policy is ___. a. policy intended to enhance public welfare b. laws passed by the legislature c. policies that are authoritative rulings related to those decisions made by government d. policy directed toward promoting the health of citizens

a. policy intended to enhance public welfare

<p>The agency whose mission is to improve the quality, safety, efficiency, and effectiveness of health care in the U.S. is ___.</p> <p>a. the Agency for Healthcare Research and Quality</p> <p>b. the Commonwealth Fund</p> <p>c. the Institute of Medicine</p> <p>d. the Organization of Nurse Executives</p>

a. the Agency for Healthcare Research and Quality

<p>The federal statute enacted in March 2010 that was designed to ensure that all Americans have access to health care is ___.</p> <p>a. the Patient Protection and Affordable Care Act</p> <p>b. the Future of Nursing: Leading Change, Advancing Health</p> <p>c. the Transition of Care Program</p> <p>d. the Patient Self-Determination Act</p>

a. the Patient Protection and Affordable Care Act

The following is not true ___. a. The nurse is accountable for selection and delegation of tasks to the delegate. b. The delegator is responsible for the actual work of the delegate. c. The delegate is fully responsible for his or her work. d. The delegator is responsible for the process.

b. The delegator is responsible for the actual work of the delegate.

>The Institute of Medicine is ___. <p>a. an organization that serves to promote health care that emphasizes access, quality, and greater efficiency, particularly for society's most vulnerable people</p> <p>b. a non-profit organization that works outside of government to provide unbiased and authoritative advice to decision makers and the public</p> <p>c. the U.S. government's principal agency for protecting the health and well-being of all Americans through the provision of essential human services, especially for those who are most needy</p> <p>d. None of the above are true.</p>

b. a non-profit organization that works outside of government to provide unbiased and authoritative advice to decision makers and the public

Creating a context of trust is the essential framework for ____. a. a place where the employee can share with you all of their problems. b. a truly safe place where conflict can be dealt with as easily and effectively as any other function, process, or relationship. c. quicker discovery of mistakes, allowing the proper punishments to be delivered more quickly. d. None of these is true.

b. a truly safe place where conflict can be dealt with as easily and effectively as any other function, process, or relationship.

All of the following are examples of elements of a professional nurse portfolio except ____. a. continuing education records b. attendance record c. committee contributions d. contributions to the community

b. attendance record

In complex adaptive systems, power is ____. a. located in the formal structure. b. closely-aligned with the point-of-care decision making. c. Both of the above are true. d. Neither of the above is true.

b. closely-aligned with the point-of-care decision making.

Times and motion studies involve _____. a. sampling activities at systematic or random intervals. b. continuous timed observations of a single person during a typical time period or shift of work. c. asking the individual to log the work performed using a data collection tool with start and stop times of each activity recorded. d. using time standards developed from past experiences.

b. continuous timed observations of a single person during a typical time period or shift of work.

A problem that confronts one, with a choice of solutions that seem or are equally unfavorable is a (n) ___. a. ethical erosion b. ethical dilemma c. ethical fading d. boundary crossing

b. ethical dilemma

All of the following are true about the ethics of care moral theory except ____. a. it is based on the insights of Carol Gilligan b. it is a process that obscures the ethical dimension of a decision c. it rejects the traditional male-centered ethics that focused on rationality, individuality, and abstract principles d. it is based on emotion, caring relationships, and concrete situation

b. it is a process that obscures the ethical dimension of a decision

All of the following are true about nursing staff skill mix except ____. a. it consists of the numbers of licensed and nonlicensed staff. b. it is determined based on who is available to work. c. it is determined based on the work that needs to be done. d. it is difficult and challenging to determine due to the multifaceted nature of patients and caregivers.

b. it is determined based on who is available to work.

You have just witnessed an incident where a patient could have been harmed by the care of another nurse. The type of ethical dilemma you are facing is a ____. a. boundary crossing b. nurse-nurse ethical dilemma c. whistle-blowing d. nurse-patient ethical dilemma

b. nurse-nurse ethical dilemma

<p>The provision of tangible recognition of professional achievement in a defined functional or clinical area of nursing is ___.</p> <p>a. the state nurse practice act</p> <p>b. nursing certification</p> <p>c. nursing licensure</p> <p>d. none of the above</p>

b. nursing certification

The Federal Register is ___. a. a policy that refers to rules that govern the workplace b. the official daily publication for rules, proposed rules, and notices of the federal government and an unbiased source of information c. a document that regulates nursing practice d. the overarching federal administrative agency concerned with quality healthcare in the United States

b. the official daily publication for rules, proposed rules, and notices of the federal government and an unbiased source of information

<p>Autonomy is all of the following except ____.</p> <p>a. the power to do a job.</p> <p>b. the right to do a job.</p> <p>c. embedded in a job description.</p> <p>d. is identified within the expectation for specific work.</p>

b. the right to do a job.

Conflict is all of the above except ______. a. normative b. to be avoided at all costs c. a metaphor for difference d. often ideological e. due to intellectual differences

b. to be avoided at all costs

The following is not true about material resources ___. a. Material resources include those resources needed for the environment and supplies and equipment to provide the required patient care. b. There is significant variation in material resources and continuing demand for new and improved material resources. c. Material resources comprise a small percentage of the cost of health care. d. All of the above are true.

c. Material resources comprise a small percentage of the cost of health care.

Complex adaptive systems theory ____. a. is grounded in work generating out of biology, mathematics, physics, and complexity science b. as applied to human behavior is represented in the study of a number of different approaches to understanding the relationship among behavior, organizations, and the larger systems that form the context. c. Both are true. d. Neither is true.

c. Both are true.

All of the following are true about evolution of interdisciplinary teams except ____. a. Digital technology has helped to create the demand for interdisciplinary teams. b. The complexity-based biopsychosocial model lends itself to health along a continuum, and therefore a role for a wide variety of clinical members. c. The physician is always the team leader and therefore guides the team. d. In this increasingly complex health environment no one discipline has all of the information, skills, and resources necessary to address the many intersecting needs of individual patients and patient populations.

c. The physician is always the team leader and therefore guides the team.

All of the following are true about price except ____. a. It is based on available dollars and the need for a good or service. b. Prices are considered from the perspective of actual costs of services. c. The seller of goods does not determine the price for the goods. d. Prices are considered from the perspective of overall expenditure of dollars.

c. The seller of goods does not determine the price for the goods.

All of the following are principles of delegation from the ANA or NCSBN website except ___. a. delegation to competent individuals. b. care coordination and the use of assistants in providing patient care. c. ensuring the integrity of the physical plant. d. the essence of professional responsibility and accountability.

c. ensuring the integrity of the physical plant.

The following is a major element of the role of dialogue ___. a. have limited breaks b. negative language is acceptable c. establish rules of speaking without interruption d. remind the parties of prior failures

c. establish rules of speaking without interruption

<p>Policies that are authoritative rulings relating to those decisions made by government are ___.</p> <p>a. social policy</p> <p>b. organizational policy</p> <p>c. public policy</p> <p>d. institutional policies</p>

c. public policy

Terms of engagement contain all of the following elements except ____. a. each member of the team will have an opportunity to speak b. avoid the use of judgment terms, such as "I agree" or "I disagree" c. team members should never use "I" statements d. the use of appreciative strategies in conversation should be encouraged and stimulated

c. team members should never use "I" statements

Ethics is ____. a. the elimination of arbitrary distinctions and the establishment of a structure of practice with a proper share, balance, or equilibrium among competing claims b. duty to keep one's promise c. the philosophical study of right and wrong action d. duty to do no harm

c. the philosophical study of right and wrong action

Issues related to the development of professionalism in the 20th century include ___. a. political equity of women b. changes in women's role expectations c. advances in women's educational opportunities d. All of the above are true.

d. All of the above are true.

The following is true about negotiation sessions ___. a. each session stands on its own merits and reflects only the activities that occurred within that session b. subsequent sessions always build on previous sessions c. early agreements on particular issues of conflict can provide the floor for subsequent clarity and agreement on more difficult issues (called chunking) d. All of the above were true.

d. All of the above were true.

Step one of the basic actions in delegation is assessing the patient or client, the situation, and the appropriateness for delegation. Important considerations within this step are ___. a. Knowing your personal delegation strengths and weaknesses as well as those of the members of your team. b. Base decisions on the needs of clients, the stability of client conditions, the complexity of the task, the predictability of the outcome, and the available resources to meet those needs and the judgment of the nurse. c. Be sure to understand the importance and expectations of the state nurse practice act, practice limitations, and job requirements. d. All of the above are considerations.

d. All of the above are considerations.

<p>Delegation is ___.</p> <p>a. required whenever there is a hierarchal order of individuals working together to accomplish goals.</p> <p>b. both a management and a legal concept.</p> <p>c. discussed in terms of authority, responsibility, and accountability.</p> <p>d. All of the above are true.</p>

d. All of the above are true.

<p>The following is true about the APRN role ___.</p> <p>a. there is currently no uniform model of regulation across the states</p> <p>b. each state independently determines the APRN legal scope of practice</p> <p>c. education, accreditation, certification, and licensure of APRNs needs to be effectively aligned to continue to ensure patient safety</p> <p>d. All of the above are true.</p>

d. All of the above are true.

A patient care delivery model ___. a. is the method or system of organizing and delivering nursing care. b. includes work delegation and resource utilization. c. emerges from the organizational mission, vision, values and structures. d. All of the above are true.

d. All of the above are true.

Bargaining is ___. a. the phase of negotiation that emphasizes the give and take related to the variety of positions at the table b. the work of exchange c. initiates the serious give and take necessary to determine which legitimate positions emerge and the degree of support for any particular position or view d. All of the above are true.

d. All of the above are true.

Core skills that must be developed and refines in order to assure successful negotiation include(s) ___. a. It is important that the individual can discern, identify, and declare specific goals for which the risk of negotiation is worth undertaking. b. There must be a certain level of flexibility and fluidity with regard to wants and needs. c. Good negotiators are well informed, clearly prepared, and understand as much about the matter under negotiation as possible. d. All of the above are true.

d. All of the above are true.

Critical elements of professional ownership include ___. a. The individual nurse values his or her specific gifts, talents, and skills and agrees to commit to the full application of them in undertaking the work of the profession. b. The individual professional member of the staff recognizes that his or her competence depends on continuous and dynamic lifelong learning. c. Competency and skill enhancement depends on the individual's commitment to membership. d. All of the above are true.

d. All of the above are true.

Establishing the framework for the negotiation and the terms of the engagement satisfies the goals of ___. a. providing a structured and positive environment as the negotiation requires b. allowing the parties to more clearly establish expectations with regard to the process c. allowing the parties to operate within the context of fairly formal rules of dialogue d. All of the above are true.

d. All of the above are true.

Examples of productivity metrics include___. a. registered nurse hours/day b. admissions, discharges, and transfers/shift c. new staff orientation hours/ total worked hours d. All of the above are true.

d. All of the above are true.

Examples of resources for ethical decision making would include ____. a. ethics committees b. ethics textbooks c. ethics journals d. All of the above are true.

d. All of the above are true.

Leadership must create an organizational context that doesn't impede membership and ownership of the work. For this to happen, the following circumstances and conditions need to be present: ___. a. Professional ownership implies that the ownership of the work belongs to the profession, not to the workplace. b. Contemporary research on the orientation of the professional/knowledge worker suggests that the traditional incentives often seen in employee-driven models do not succeed to motivate the action of this worker. c. Leaders must on-board new professionals through the use of well-identified professional processes, such as granting privileges instead of hiring, and peer-based selections of incoming members instead of management hiring practices. d. All of the above are true.

d. All of the above are true.

Performance evaluations review the worker within the context of the capacity to function and ___. a. the ability to do the work b. the ability to get the job done c. the ability to get along with others who are doing their jobs d. All of the above are true.

d. All of the above are true.

Reasons why shared governance organizational structure operates effectively in complex adaptive systems like the healthcare environment include ___. a. The sustainability of healthcare services fully depends on how the point-of-service professionals function individually and collectively. b. The emergence of knowledge organizations demonstrates that knowledge is not fixed or finite and does not operate simply as a capacity. c. In a continuum of care approach, no one healthcare organization can own, control, or unilaterally mandate all of the service linkages and connections necessary to fully serve a specific population. d. All of the above are true.

d. All of the above are true.

Sources of ethical dilemmas include ____. a. values b. diversity c. political views d. All of the above are true.

d. All of the above are true.

Team progress related response strategies that the team leader could initiate within the team process are all of the following except ____. a. reaffirm specific goals and activities that were identified as essential to the team's progress toward fulfillment of the purpose of the team b. place the problem, barrier, or challenge specifically within the context of particular stages or elements of change management. c. when the context of the issue has been delineated, help break the problem down into smaller issues d. All of the above are true.

d. All of the above are true.

The central characteristics of accountability that are nonnegotiable for the profession represent the following principles ___. a. Accountability generally reflects the individual professional commitment to performance and to action. b. Accountability demands a dynamic convergence between ownership, action, impact, and outcome. c. Individual accountability cannot be sustained if the system structure does not allow it to be legitimately expressed by those who own it. d. All of the above are true.

d. All of the above are true.

The following are strategies for assessing one's time management ___. a. Determine what your personal abilities are in managing your work across s span of time. b. Take into consideration both your own personal assessment and the feedback from others on your team. c. Personal reflection specific to meeting deadlines, feeling stressed or overwhelmed, and managing interruptions are areas that should be considered. d. All of the above are true.

d. All of the above are true.

The following are true about productivity ____. a. It is a ratio comparing what is produced to what is required to produce it. b. It is a measure of output from a production process, per unit of input. c. Aggregate productivity ratios are helpful in determining the overall efficiency of processes and individuals at a macro level. d. All of the above are true.

d. All of the above are true.

The following is information to be gathered by the team leader prior to assigning roles _____. a. discerning the character of the work b. discerning the distribution of the work c. determining the best way to break down elements of the work into meaningful components. d. All of the above are true.

d. All of the above are true.

The following is true ---. a. The relationship between the individual and the workplace unfolds through the lens of job categorization. b. The job is the contextual framework for the relationship between the individual and the workplace. c. Creating a job context for work cannot produce professional outcomes. d. All of the above are true.

d. All of the above are true.

The impact of technology on the team process means that ____. a. communications become available to team members more quickly b. communication may be more succinct c. body language and emotions are not available to factor in with electronic communication d. All of the above are true.

d. All of the above are true.

The process of recruiting nurses ___. a. involves identifying and selecting individuals who closely match the needs of the unit and organization b. involves defining or anticipating needs for services c. includes advertising, interviewing, selecting, and hiring d. All of the above are true.

d. All of the above are true.

The role of the leader in an era of change is ___. a. helping the staff to anticipate change b. translate change into a language that colleagues can understand c. actively engage in change d. All of the above are true.

d. All of the above are true.

The sense of lack of justice comes from what is considered ___. a. unfair work conditions b. unfair procedures c. unfair relationships d. All of the above are true.

d. All of the above are true.

Value is about ___. a. doing the right work rather than doing a good job b. work that is informed by purpose c. the product of a measured and defined relationship between effective process and meaningful outcome d. All of the above are true.

d. All of the above are true.

Effective delegation ___. a. distributes the workload equitably b. distributes the workload effectively c. enhances collaboration with other healthcare providers d. All of the above is true.

d. All of the above is true.

Nurses are encouraged to avoid relationships in which individuals are known to have the following characteristics ____. a. bully and harass others b. engage in routine gossip c. be unreliable and need constant reminders to complete work d. All of the above should be avoided.

d. All of the above should be avoided.

<p>Considerations in being "media ready" include ___.</p> <p>a. practicing with internal public relations staff</p> <p>b. avoiding off-the-record comments</p> <p>c. defining key messages and staying on message</p> <p>d. All of the above.</p>

d. All of the above.

<p>Policies are generated from ___.</p> <p>a. state legislatures</p> <p>b. state agencies authorized by legislation</p> <p>c. state professional associations</p> <p>d. All of the above.</p>

d. All of the above.

Patient workforce management includes five highly interconnected and interrelated components, which include all of the following except ____. a. Establishment of a patient care delivery model. b. Patient needs and nurse interventions identification. c. Creation of a core staffing schedule to support patient needs. d. Daily staffing process that is consistent each day. e. Evaluate value and outcome.

d. Daily staffing process that is consistent each day.

<p>All of the following are true about accountability except___.</p> <p>a. It is being answerable for actions or inactions of self or others.</p> <p>b. It is the obligation to account for or explain the events.</p> <p>c. There is individual accountability and organizational accountability for delegation.</p> <p>d. It describes the distribution of work that each staff member is to accomplish in a given period of time.</p>

d. It describes the distribution of work that each staff member is to accomplish in a given period of time.

All of the following are true about a shift to the health model except ___. a. The health model requires a network. b. A preventive health service structure is key. c. Patients must access services earlier. d. Patients are seen only when ill.

d. Patients are seen only when ill.

All of the following are true about the team leader and their role except ___. a. It is the role of the team leader to both monitor and moderate the effect of progress and challenges to an effective team dynamic. b. The team leader must not be overwhelmed by the seesaw between small successes as they intertwine with moments of challenge. c. The team's small successes should be celebrated by members as they progress. d. The leader should never point out signs of progress until a goal is totally achieved.

d. The leader should never point out signs of progress until a goal is totally achieved.

The purpose for creating teams is all of the following except ___. a. to provide a general framework for care delivery b. to advance the capacity of professionals to fully engage particular planning activities c. to guide problem solving in a way that addresses patient care and improves team effectiveness d. To make staffing ratios easier to calculate

d. To make staffing ratios easier to calculate

<p>The committee of Future of Nursing: Leading Change, Advancing Health developed the following key messages ___.</p> <p>a. Nurses should practice to the full extent of their education and training.</p> <p>b. Nurses should achieve higher levels of education and training through an improved education system that promotes seamless academic progression.</p> <p>c. The diploma model for nurses training should be revived.</p> <p>d. a and b</p>

d. a and b

Healthcare economics is ___. a. a branch of economics focused on efficiency, effectiveness, and behavior in production and consumption of healthcare goods and services. b. the study and science of how human needs are perceived in relationship to what supply is available. c. a topic that only managers need to be aware of. d. a and b

d. a and b

Responsibility ___. a. is embedded in work b. sees the individual from the perspective of the work c. assumes some level of intensity of the individual and any investment and ownership of the role, tasks, and activities d. a and b e. All of the above are true.

d. a and b

The following are included in the guidelines for interaction in negotiation ___. a. never say "never" b. make many concessions early in the negotiations c. do listen carefully to all parties d. a and c e. All of the above are true.

d. a and c

12. Steps in handling conflict include ___. a. identifying the problem b. immediately reporting the problem to hospital administration c. making sure issues are expressed d. a and c are true.

d. a and c are true.

Bioethics is__. a. the sum total of individual and collective experience, knowledge, and good sense b. guidelines for behavior specific to a moral framework for professional practice c. supporting and promoting patients' healthcare rights and enhancing community health and policy initiatives that focus on the availability, safety, and quality of care d. a subdiscipline of applied ethics that studies questions surrounding biology, medicine, and the health profession

d. a subdiscipline of applied ethics that studies questions surrounding biology, medicine, and the health profession

8. Validity measures___. a. the extent to which data is reproducible. b. the extent to which a workforce management system measures what it is designed to measure. c. the ability to quantify and/or predict patient needs for nursing care. d. b and c e. All of the above are true.

d. b and c

The primary point of accountability is ___. a. to be responsible for a stated task. b. to be able to demonstrate impact and results. c. to achieve long-term viability. d. b and c

d. b and c

14. Values conflicts ______. a. are the easiest to address in the workplace b. are difficult to resolve because they relate to who people are c. are difficult to resolve because they relate to what people believe d. b and c are true

d. b and c are true

15. Areas to include in the mediation process as relationship issues are explored include all of the following except ___. a. mutual respect b. needs versus wants c. compassion and empathy d. blaming others.

d. blaming others.

Nursing career management includes all of the following except ___. a. maintaining a current license b. maintaining continuing competence c. participating in professional organizations d. exploring alternative careers

d. exploring alternative careers

Step three in the delegation process is planning for the work to occur. The following are steps which provide guidelines for preparing with the delegate ____. a. Clearly identify the work that needs to be done. b. Clearly identify the importance of requesting assistance when needed. c. Create a plan for daily feedback between the delegator and delegate specific to delegation principle performance. d. Discuss strategies to increase effectiveness of delegation and supervision activities, including frequency of supervision and potential for additional delegation. e. All of the above are steps.

e. All of the above are steps.

10. Alienation____. a. is encouraged by permitting the reflective work of the profession to be sacrificed by unrelenting work expectations. b. is encouraged when team members are kept busy doing without the opportunity to think about what they're doing. c. that flourishes prevents individuals from ever arriving at that unique fellowship and strong sense of professional relationship that is a sign of a committed team. d. causes members of the team to give up participating. e. All of the above are true.

e. All of the above are true.

<p>Activities which all nurses should embrace as they become more involved in the profession include ___.</p> <p> </p> <p>a. becoming competent in the processed of policy development and the differing approaches that are effective at the local, state, and national levels</p> <p>b. developing focused partnerships with stakeholders who know and understand the evidentiary support for policy making or changing</p> <p>c. taking a risk and proposing a new policy that is of great personal interest</p> <p>d. examining a current policy to identify its effectiveness</p> <p>e. All of the above are true.</p>

e. All of the above are true.

All of the following are true about accountability in shared governance except ___. a. Accountability drives the work of professionals. b. Accountability is the foundation for all knowledge work performance. c. Every member of a complex adaptive system has both rights and obligations that generate from the full participation in the life and activities of the system. d. Every member of the system must fully contribute to the extent of his or her capacity in a way that ensures that the system thrives and fulfills its essential value. e. All of the above are true.

e. All of the above are true.

Applying for a new position leads to the following changes ___. a. It changes the individuals you socialize with. b. It changes your abilities to manage policies and practices. c. It changes your salary and benefits. d. It changes your work location. e. All of the above are true.

e. All of the above are true.

Approaches to staffing decisions include ___. a. nurse-patient ratios b. patient classification systems c. legislative requirements d. different skill levels e. All of the above are true.

e. All of the above are true.

Collaboration ___. a. is the essential central methodology through which the diverse team stakeholders work b. advances the interactions that define the team's response to patient needs c. advances the team's competence in serving their population well d. implies the capacity to have team members work well together e. All of the above are true.

e. All of the above are true.

Embedded in the fear of conflict are issues of all of the following except ____. a. personal security b. identity c. safety d. relational integrity e. All of the above are true.

e. All of the above are true.

Examples of interventions to address the gap between needs and actual staffing are the following ___. a. reevaluating patient acuity ratings. b. postponing admissions. c. postponing nonemergent patient care, d. floating existing staff to the unit of need. e. All of the above are true.

e. All of the above are true.

Issues that impact ethics in health care include ___. a. the values of the patient b. the values of the caregivers c. errors in the clinical setting d. moral courage e. All of the above are true.

e. All of the above are true.

Knowledge workers (professionals) ___. a. a need a very specific organizational context in order to facilitate good practice and positive outcomes b. are intrinsically motivated c. see their work as more than simply a job d. a and b e. All of the above are true.

e. All of the above are true.

Mentors and coaches address ___ with the mentored nurse. a. professional role behaviors b. appearance c. attitude d. conflict management e. All of the above are true.

e. All of the above are true.

Nonverbal signs the leader must be able to read include ___. a. body language b. gestures c. eye movements d. facial expressions e. All of the above are true.

e. All of the above are true.

Nursing is ____. a. a scientific process founded on a professional body of knowledge. b. a learned profession based on an understanding of the human condition across the lifespan. c. an art dedicated to caring for others. d. a dynamic discipline that increasingly involves more sophisticated knowledge, technologies, and client care activities. e. All of the above are true.

e. All of the above are true.

Settings where nursing opportunities may be found include all of the following except ___. a. healthcare organizations b. community centers c. outpatient settings d. insurance industry e. All of the above are true.

e. All of the above are true.

Significant shifts underway in health care include ___. a. Tertiary care is slowly being deconstructed. b. Digital technology is making it possible to create an increasingly portable diagnostic, therapeutic, and interventional environment making it increasingly mobile, fluid, flexible, and portable. c. Systems compete for value and quality. d. The fundamental value of practice is no longer embedded in volume measures related to how much an individual has done but in what difference and impact action had. e. All of the above are true.

e. All of the above are true.

Structure in the healthcare system that best supports healthcare leadership is ____. a. resource-effective b. timely c. value-based d. works to advance the mission and vision of the healthcare system e. All of the above are true.

e. All of the above are true.

The early identification of conflict is important to help prevent escalation. Many stressors are manifested in particular modes of ______. a. communication b. relationships c. interactions d. expression of personal emotions e. All of the above are true.

e. All of the above are true.

The following is true about conflict and conflict management ___. a. Principles of conflict management are essential to navigating the landscape of transformation and change. b. Conflict is a normative dynamic. c. A good conflict process includes identifying conflict in its earliest stages. d. Conflict resolution and mediation is a learned and developed skill that comes with practice and application. e. All of the above are true.

e. All of the above are true.

The leader needs to be comfortable with conflict. Factors which are important to this occurring are the following: a. The leader must be comfortable with self-expression of feelings, personal insights, and challenges to prevailing views and circumstances. b. The leader needs an environment that is not constraining or controlling or creates structural barriers that make it difficult to express personal feelings. c. The good leader embraces his or her own feelings regardless of intensity. d. The leader must assess their level of comfort and self-expression in the presence of conflict or conflict situations. e. All of the above are true.

e. All of the above are true.

Transition to nursing practice is sometimes difficult because ____. a. an adequate support system for new graduates may not be in place. b. subtle hazing may occur to a new employee. c. new ideas they present are quickly dismissed and discouraged. d. a and b are true. e. all of the above are true.

e. All of the above are true.

Criteria for assessing and monitoring meaningful measures includes ____. a. Prioritize consumer and purchaser needs. b. Focus measurement on areas where there is a potential for improvement in outcome quality. c. Require that all patients fitting clinical criteria be included in analysis. d. Measure provider performance at all levels. e. All of the above is true.

e. All of the above is true.

Policy making takes place ___. a. at the national level. b. at the state level c. at the local level d. within agencies e. All of the above is true.

e. All of the above is true.

Major resources in a healthcare organization include ____. a. fiscal support b. physical setting c. supplies d. technology e. All of the above.

e. All of the above.

The multistate licensure ___. a. allows a nurse to have one license in a primary state of residence and be able to practice in other states with multistate statutes. b. increases the cost of licensure fees when nurses practice in more than one state. c. is available in 24 states. d. is available in all states. e. a and c are true. f. b and d are true.

e. a and c are true.

<p>Critical thinking takes ___ into consideration when deciding what to believe or do.</p> <p>a. focus</p> <p>b. reasoning</p> <p>c. assignments</p> <p>d. implications</p> <p>e. a, b, and d</p> <p>f. All of the above are true.</p>

e. a, b, and d

Risk is ___. a. a sign of the continuous and dynamic reach of the human experience in new directions in new ways b. exploring new territory in a way that advances the human experience and improves it in important ways c. to be avoided in the healthcare arena d. present at some level with all meaningful change e. a, b, and d f. All of the above are true.

e. a, b, and d

The underlying reasons for advancing to management and leadership positions are ____. a. the desire to advance quality practices b. to advance excellence c. because of a dislike for the clinical role d. to introduce new ideas for a better work environment e. a, b, and d f. All of the above are true.

e. a, b, and d

Major factors that are critical to the effectiveness of the team leader's role relate specifically to ____. a. the adequate structuring of the team b. the type of health care setting c. the use of good process d. the progress of the team toward its goals or in fulfillment of its purpose e. a, c, and d f. All of the above are true.

e. a, c, and d

The leader completely orchestrates and manages ____ associated with the negotiation. a. the atmosphere b. the discussions c. the space d. the processes e. a, c, and d f. All of the above are true.

e. a, c, and d

Informatics and related technology include ___. a. researching new medications b. designing complex documentation and monitoring systems c. electronic monitoring of patients from remote locations d. a and b e. b and c f. All of the above are true.

e. b and c

Areas of focus necessary to get the work done in healthcare settings include a system that requires ____. a. clear delineation and assignment of work based on skills. b. licensure and competence. c. recruitment and retention of competent caregivers. d. assurance of value-based outcomes. e. integration of the work of all caregivers into a whole that meets the needs of patients. f. All of the above are true.

f. All of the above are true.

The expert opinion or expert panel technique to measure work _____. a. identifies time requirements for certain work. b. is a consensus approach. c. uses professional judgment to determine staff required. d. provides a flexibile approach that focuses on a critical review of nursing practice. e. a and d f. All of the above are true.

f. All of the above are true.

Accountability needs ____ for it to be expressed. a. autonomy b. responsibility c. authority d. competence e. a, b, and c f. a, c, and d

f. a, c, and d

Legislative issues could impact the following ethical issues ___. a. boundary violations b. betrayal c. code of ethics d. rationalization e. a, b, and c f. a, c, and d

f. a, c, and d

Collective bargaining is protected by ___. a. the Department of Commerce b. the Nation al Labor Relations Act c. the Taft-Hartley Act d. Public Law 93-360 e. a and b f. b, c, and d g. All of the above.

f. b, c, and d

For complex adaptive systems to thrive, the following principles must operate ___. a. The whole is never greater than the parts. b. A problem in one part of the system ultimately affects the whole system. c. All disciplines serve the user and/or serve someone who serves the user. d. In complex adaptive systems, form and function interact with each other, otherwise they are not a dependent relationship. e. a, b, and d f. b, c, and d g. All of the above are true.

f. b, c, and d

The following are phases of negotiation ___. a. readying for negotiation b. establishing the framework for negotiation c. intensive interaction d. bargaining e. closing the interaction f. a and c g. All of the above.

g. All of the above.

The five types of conflicts are ___. a. relationship conflicts b. information conflicts c. interest conflicts d. organizational conflicts e. values-based conflicts f. conflicts with administration g. a through e are true h. All of the above are true.

g. a through e are true

11. Five personal barriers that have a dramatic impact on the individual leader's ability to understand, engage, and manage conflict include ____. a. willingness to confront conflict b. uncertainty about the capacity to handle the conflict emotionally and personally c. recognizing that most people's attitude toward conflict is a negative one d. lack of skill e. lack of exposure and experience f. previous family conflicts experienced g. a, b, c, d, and e are true. h. All of the above are true.

g. a, b, c, d, and e are true.

The foundations of shared governance include ___. a. ownership b. accountability c. vertical structures d. partnership e. equity f. a, b, and c g. a, b, d, and e

g. a, b, d, and e


Ensembles d'études connexes

US History Finals Practice Questions

View Set

marketing chapter 17: integrated marketing communications

View Set

Sports Nutrition Final Study Set

View Set

FINN 3003 NOEL MORRIS FINAL EXAM

View Set

Europe Moodle Quiz, South Asia Moodle Quiz, East Asia Moodle Quiz Review, South East Asia Moodle Quiz

View Set